1. Trang chủ
  2. » Ôn thi đại học

Phương pháp giải phương trình hàm trên tập rời rạc

98 17 0

Đang tải... (xem toàn văn)

Tài liệu hạn chế xem trước, để xem đầy đủ mời bạn chọn Tải xuống

THÔNG TIN TÀI LIỆU

Thông tin cơ bản

Định dạng
Số trang 98
Dung lượng 6,57 MB

Nội dung

Trường hợp 1. Từ đây ta thử lại đề bài và thấy thỏa mãn nên ta hoàn thành bài toán. Đây là một bài toán phương trình hàm trên tập rời rạc khó và điều kiện ràng buộc khá là khó chịu. Và [r]

(1)

NGUYỄN MINH TUẤN DỖN QUANG TIẾN TƠN NGỌC MINH QUÂN

PHƯƠNG TRÌNH HÀM TRÊN TẬP RỜI RẠC

(2)

Littited Edition

Chuyên đề Bồi dưỡng Học sinh giỏi

Phương trình hàm Trên tập rời rạc

(3)

Copyright © 2019 by Tap chi va tu lieu toan hoc

(4)

TẠ

P CH

Í VÀ TƯ

LI

ỆU TOÁN

H

C

H

C

H

C

Nhng bi toỏn phỵng trỡnh hàm ngày trở nên phổ biến bän học sinh u Tốn chúng xut hin thỵng xuyờn cỏc thi hc sinh gii cỏc cp cỹng nhỵ kỡ thi chn i tuyn quốc gia, VMO hay kì thi khu vực quc t m ta ỵc bit n c bit, cỏc lp dọng phỵng trỡnh hm, thỡ dọng phỵng trỡnh hm trờn cỏc ri rọc l mt mõng ỵc học sinh ý tới độ khó v chỵa ỵc tip xỳc nhiu ng thi ngoi vic s dýng cỏc kù thut x lý phỵng trỡnh hm bân cịn phâi sử dýng tính chất số học đặc sắc cûa tập rời räc nhỵ l: tớnh chia ht, tớnh cht cỷa s nguyờn t, cỷa s chớnh phỵng, Trong ebook ny chỳng mang tới cho bän đọc tuyển tập bi toỏn phỵng trỡnh hm trờn ri rọc v mt s bi toỏn phỵng trỡnh hm khỏc hay v khó với lời giâi vơ đặc sắc nhằm giúp bän đọc có nhiều cách nhìn khác v mõng toỏn ny ng thi cỹng nhỵ chun b cho kì học sinh giỏi, olympic

Mình xin gửi lời câm ơn tới

1 Thầy Huỳnh Kim Linh – THPT chuyên Lê Quý Đôn – Khánh Hịa – Đã góp ý giúp bọn phần ni dung

2 Bọn La Th ụng Phỵng Đäi học Hoa Sen – Đã giúp bọn chỵnh sửa bân thâo đề hoàn thiện

Một lần gửi lời câm ơn bän, thầy cô ûng hộ theo dõi fanpage suốt thời gian qua Hy vng ebook ny s giỳp ớch ỵc cho mi ngỵi Thank you!

Nhúm tác giả Nguyễn Minh Tuấn Doãn Quang Tiến Tôn Ngọc Minh Quân

(5)

PHƯƠNG TRÌNH HÀM TRÊN TẬP RỜI RẠC TẠP CHÍ VÀ TƯ LIỆU TỐN HỌC

(6)

CH

INH

PH

C

OL

YM

PIC T

O

ÁN

Chuyên đề

PHƯƠNG TRÌNH HÀM TRÊN TẬP RỜI RẠC

Tạp chí tư liệu toán học Để giải tốn phương trình hàm tập rời rạc mà giải tính chất số học nên lưu ý đến số dấu hiệu sau:

 Nếu xuất biểu thức tuyến tính chứa lũy thừa, nghĩ đến tốn liên quan đến cấp phần tử, phương trình đặc biệt phương trình Pell hay phương trình Pythagore,<hay đưa việc xử lý phương trình vơ định nghiệm ngun

 Nếu hàm số cho hàm nhân tính, ta thường hay xét đến giá trị hàm số điểm số nguyên tố dãy vô hạn số nguyên tố

 Sử dụng đẳng thức bất đẳng thức số học

 Và đặc biệt nhất, số toán, hệ số đếm dùng để xây dựng nhiều dãy số có tính chất số học thú vị Trong hệ số 10 khó nhận quy luật dãy, chọn hệ số phù hợp tốn giải đơn giản nhiều

Nếu g2,g ,với g số đếm, số nguyên dương M biểu diễn dạng:

 

  1 2  

1 ng n n n n

M a a a a g a g a g a với 1a1  g 1;0    ai g 1, i 2, n

Cơ số đếm mà hay sử dụng tốn phương trình hàm tập rời rạc

(7)

TẠ

P

CH

Í VÀ

LI

ỆU TO

ÁN

H

C

I ĐỀ BÀI

Câu Tìm tất hàm số f :  thỏa mãn điều kiện sau:

      

3f n 2f f n n n,

Câu Tìm tất hàm số f :  thỏa mãn điều kiện sau

m n f m   n2mf n nf m ,m n,  1 

Câu Cho hàm số f : *  * thỏa mãn điều kiện sau:

 1   ,  *

f n f f n n

Chứng minh f n   n n, *

Câu Tìm tất hàm số f : *  * thỏa mãn điều kiện sau:

     

    

2 , , *

x f y f x y x y

Câu Tìm tất hàm số f : *  * thỏa mãn điều kiện sau:

     2    

2 , , * *

f m f n m n m n

Câu Tìm tất hàm f :  thỏa mãn tồn số k số nguyên tố p cho

với n k f n p ,    f n  m n f m 1  f n 1

Câu Cho p số nguyên tố lẻ Tìm tất hàm f :  thỏa mãn đồng thời

điều kiện:

i) f m  f n với m n modp

ii) f mn  f m f n   ,m n, 

Câu Tìm số ngun khơng âm n nhỏ cho tồn hàm số f : 0, khác

hằng số thỏa mãn đồng thời điều kiện:

i) f xy  f x f y   ,x y, 

ii) 2f x 2y2 f x  f y 0,1, , ,n x y, 

Với số n tìm được, tìm tất hàm số thỏa mãn

Câu Giả sử hàm số f : *  thỏa mãn điều kiện sau:

 1 1

f  

      

 

  

 

 

    

  

1

1 2

1 2

n

f if n m

f n

n

f if n m

Tìm giá trị n cho f n 2019

(8)

CH

INH

PH

C

OL

YM

PIC T

O

ÁN

       

     

     

 

 

 

   

    

1 1, 3

4 2 3 2

f f

f n f n

f n f n f n

f n f n f n

với số nguyên dương n

Câu 11 Cho hàm số f :    thỏa mãn đồng thời điều kiện:

f n  ước n2018 với n

f a f b     f c  với a b c, ,   a2b2 c2

a) Chứng minh n lẻ n f n 1

b) Gọi A tập hợp giá trị có f 2  f2018 Tính A

Câu 12 Có tồn hàm số f S: Sthỏa mãn điều kiện

     2 , ,  , 

f a f b f a b a b S a b khơng, S *\ 1 ?

Câu 13 Tìm tất hàm số f : *  * thỏa mãn điều kiện

  2       2   *

1 ,

n f n f f n n n n

Câu 14 Tìm tất hàm số f :  thỏa mãn đồng thời hai điều kiện sau:

i) x f y f x     y f x f y    với x y,  ;

ii) Tập hợp        

 

 , , , 

f x f y

I x y x y

x y khoảng

Câu 15 Tìm hàm số f : *  * thỏa mãn f m2  f n m  n2,m n,  *

Câu 16 Cho hàm f x y ,  thỏa mãn điều kiện:

f 0,y  y 1;f x 1,0 f x ,1  f x 1,y1 f x f x ,  1,y

Với số ngun khơng âm x y, Tìm f4,1981?

Câu 17 Cho hàm f :   thỏa mãn điều kiện sau:

i) f n 1 f n ; n

ii) f f n  3 ,n n Z  

Hãy tính f2003 

Câu 18 Cho f n  hàm số xác định với n * lấy giá tị khơng âm thỏa mãn tính

chất:

 n m,  *: f m n   f m  f n 

lấy giá trị  f 2 0 f  3 0

(9)

TẠ

P

CH

Í VÀ

LI

ỆU TO

ÁN

H

C

Tính f 2000

Câu 19 Cho f g, hàm xác định thỏa mãn điều kiện

     2    , , 

f x y f x y f x g y x y

Chứng minh f x 0 f x    1, x g y 0  a

Câu 20 Cho hàm số f :  thỏa điều kiện

i) f x    1 x x;

ii) f x y   f x f y   ;x y, 

Chứng minh tồn hai số a b;  mà f a f b    0

Câu 21 Cho  , y 2003cos 2   cos   

2

f x x y a x y với a, 

Chứng minh minf x y , 2 maxf x y , 2 2003

Câu 22 Cho hàm số     

2 1

,

x

f x x

x

Giả sử f x0 x f xn   f fn1 x     n *, x

Chứng minh  

 

1

1

, 1,0,1

1

n

n n

f x

n x

f x x

f x

      

 

  

 

Câu 23 Cho hàm số f : * *  * hàm số thỏa mãn đồng thời điều kiện sau:

i) f  1,1 2

ii) f m 1,n f m n , m m n, ,  *

iii) f m n , 1 f m n ,  n m n, ,  *

Tìm tất cặp số  p q, cho f p q , 2019

Câu 24 Tìm tất hàm số f :  thỏa mãn điều kiện sau:

i) 0 f x x2, n

ii) f x  f y  chia hết cho x y với x y,  ,x y

Câu 25 Tìm tất hàm số f : *  * mà tập * xx0 thỏa mãn:

   2     , ,  * 1 

f xy

f x f y xyf xy x y

f x y

Câu 26 Cho hàm f :   hàm số thỏa mãn với n1 có số ngun

tố p ước n cho:           

n

f n f p f

p

  32018  52019  720202017.

(10)

CH

INH

PH

C

OL

YM

PIC T

O

ÁN

Hãy tính giá trị biểu thức G f 20182018  f 20192019  f 20202020

Câu 27 Tìm tất hàm số f : *  * thỏa mãn:

            

3 2 2 *

2f m n f m f n f m f n , m n,

Câu 28 Giả sử f :  hàm liên tục giảm cho với x y,   ta có

           

f x y f f x f y f y f x

Chứng minh f f x  x

Câu 29 Cho song ánh f :  Chứng minh tồn vô số a b c, ,  với a b c, , 

thỏa mãn a b c  2f b  f a  f c 

Câu 30 Có hàm f : * * thoả mãn đồng thời điều kiện sau

a) f 1 1

b) f n f n   29f n 121997,  n *

Câu 31 Tìm tất hàm số *  *

:

f cho

a) f 2 2

b) f m n  f m f n    với m n,  *, UCLN m n , 1

c) f m    f n   * 

, ,

m n m n

Câu 32 Tìm tất hàm số f :  thỏa mãn

         1, , 

f m n f mn f m f n m n

Câu 33.Tìm tất hàm số f :  thỏa mãn f 0 2

 

  2     2  , , 

f x f x y f x f y x y  1

Câu 34.Tìm tất hàm số f :  cho f f n   f n 2n  3, n  1

Câu 35.Chứng minh tồn hàm số f : * * thỏa mãn

 

      , ,  *  

f m f n n f m b m n b  i

Câu 36.Hãy xác định tất hàm số f : *  * thỏa mãn đẳng thức:

   1  2   3

f n f n f n f n a 1

Với a số tự nhiên thỏa mãn a1 số nguyên tố

Câu 37.Tìm tất hàm số f : *  * thỏa mãn f nt   a   f nan t a k với

     

t

t

f nf f f n với a t, số tự nhiên tùy ý thỏa mãn k t2 1 a

Câu 38.Cho hàm số f :  thỏa mãn:

   

         

   

2 2

,

f n f n f n f n f n

n

f n f n

        

 

(11)

TẠ

P

CH

Í VÀ

LI

ỆU TO

ÁN

H

C

Tìm n cho f n 2009

Câu 39.Tìm tất hàm số f :  thoả mãn:

       

1 1, , ,

3 f xy 3 f xzf x f yz  x y z

Câu 40.Cho n n2 hàm số f :  cho:

nn     ; ,

f xyxf x f f yx y  *

a) Giả sử f 20020 Tính f 2002  b) Tìm hàm số f

Câu 41 Tìm tất hàm số f :  thỏa mãn

 3   2  3  , ,

f x y zf xf yf zx y z

Câu 42.Cho hàm số f : *  * thỏa mãn đồng thời hai điều kiện:

a) f ab  f a b f a b ,   ,  với a b,  *,a b ;    a b, , a b, bội chung nhỏ nhất, ước chung lớn hai số nguyên dương a b, ;

b) f p q r    f p  f q  f r  với số nguyên tố p q r, ,

Tính giá trị f 2013? Kí hiệu * tập hợp tất số nguyên dương

Câu 43 Đặt Ff : 0,1    0,1 n2 Tìm giá trị nhỏ c thỏa mãn điều kiện

   

1

0

n

f x dx c f x dx

 

Với f Ff hàm liên tục

Câu 44 Tìm tất hàm f : 1,1  liên tục, thỏa mãn:

  2

x f x f

x

 

   ,  x  1,1

Câu 45 Có thể tồn hay khơng hàm số f :  , liên tục thỏa mãn điều

kiện: Với số thực x, ta có f x  số hữu tỉ f x 1 số vô tỉ

Câu 46 Tìm tất hàm số f :  thỏa mãn điều kiện f x  f t  f y f z  với

mọi số hữu tỉ x y z t   x y z t, , , theo thứ tự lập thành cấp số cộng

Câu 47 Giả sử r s,  hai số cho trước Tìm tất hàm số f :  thỏa mãn điều

kiện f x f y    f x r     y s x y, ,  ?

Câu 48 Tìm tất hàm số f :  cho với tất số nguyên a b c, , thỏa mãn

0

a b c   , đẳng thức sau đúng:  

 2   2   2            

2 2

f af bf cf a f bf b f cf c f a

Câu 49 Tìm tất hàm f g, :   có đạo hàm  thỏa mãn

   

' g x

f x

x

  ; g x'  f x 

x

(12)

CH

INH

PH

C

OL

YM

PIC T

O

ÁN

Câu 50 Tìm tất hàm f : *  có đạo hàm * thỏa mãn

      , *

f xyf xf yx y  1

Câu 51 Tìm tất hàm f :  thỏa mãn

 

  1 

f f n    n b n

trong b số nguyên dương chẵn

Câu 52 Tìm tất hàm f :   thỏa mãn:

i) f xf y  yf x  ,x y   1

ii) lim  

x f x

Câu 53 Chứng minh tồn song ánh f :   cho

3          ,

f mn m n   f m f nf mf nm n 

Câu 54 Tìm tất hàm f :  thỏa:

 

 

      

3f f f n 2f f nf n 6 ,n  n

Câu 55 Tìm tất hàm số f : 0;   0; thỏa mãn điều kiện:

 

     , 0;   

f f xyf yf xx y 

Câu 56 Chứng minh tồn hàm số f xác định tập số thực

dương, nhận giá trị thực dương thỏa mãn f f x  6x f x  

Câu 57 Hàm số f :  thỏa mãn đồng thời điều kiện sau:

      

       

     

: , : 2 ,

:

i f f n n n

ii f f n n n

iii f

  

    

Tìm giá trị f1995 , f 2007

Câu 58 Tìm f : 0,1  thỏa mãn f xyz xf x yf y zf z  x y z, ,  0,1

Câu 59 Tìm tất hàm f xác định thỏa mãn đồng thời điều kiện sau:

           

2 ,

1

f n f k n f k n f n f k k n f

    

 

 

Câu 60 Tìm tất hàm số f : *  * thỏa mãn đồng thời hai điều kiện sau:

 

 

   

* *

*

2 , ,

1 ,

f f n n k n k

f n f n n

     

 

   



Câu 61 Tìm tất hàm số f :  thỏa mãn đồng thời hai điều kiện sau:

   

 

2013 2016 4,

f

f f n n n

 

    



*

:

(13)

TẠ

P

CH

Í VÀ

LI

ỆU TO

ÁN

H

C

       ,  1

f nf n  f nf n  n

Câu 63 Tìm tất hàm f :   thỏa mãn:

 

     

f x f y  f x y  f y

Câu 64 Tìm số nguyên dương m nhỏ cho tồn hàm số f : *  \ 1;0;1 

thỏa mãn đồng thời điều kiện sau i) f m  f 2015 , f m1 f 2016 ;

ii)    

  11, 1,2,

f n

f n m n

f n

  

Câu 65 Xác định hàm số f x  liên tục   thỏa mãn đồng thời điều kiện:

f 2x 2f x  với x ,  1

f f x e 3  f x 1x e2 x 1f x 

với x ,  2  f e 1  e 1  f ,  3

f k  số nguyên dương với số nguyên dương k,  4

Câu 66 Tìm tất hàm f:  thỏa mãn đồng thời hai điều kiện sau:

 Với cặp a, b nguyên dương không nguyên tố nhau, có f a f b     f ab   Với a, b nguyên dương tồn tam giác khơng suy biến có độ dài ba

cạnh f a f b   , f a b  1

Câu 67 Tìm hàm số f : 1;   thoả mãn điều kiện:

       

f xf yy x f xy với x y, 1 1

Câu 68 Tìm tất hàm f : *  * thỏa mãn đẳng thức:

   

 2  2

f f mf nmn , với m n,  *

Câu 69 Tìm tất số ngun khơng âm n cho tồn hàm f : 0; khác

hằng thỏa mãn đồng thời điều kiện sau

i) f xy  f x f y   ,x y, 

ii) 2f x 2y2 f x  f y x y  ,  0;1; 2; ; n

Câu 70 Tìm tất hàm số f : * * thoả mãn điều kiện:

 

 2 2 3 2    2    *

2 f mnf m f nf n f m ,m n, 

Câu 71 Tìm tất hàm số f :  thoả mãn điều kiện:

f 0 c

    

 

3

1 , *

3

f n

f n n

f n

   

(14)

CH

INH

PH

C

OL

YM

PIC T

O

ÁN

Câu 72 Tìm tất hàm số f :  thỏa:

 2      ,

f af bf f a b a b

Câu 73 Có tồn hay không hàm số f :  cho

 

    , ,  1

f m f n  f m  n m n

Câu 74 Cho hàm số f :  hàm số thỏa mãn điều kiện sau:

i) f mn  f m f n   ,m n, 

ii)m n  ước f m  f n  với m n, 

Chứng minh tồn số tự nhiên lẻ k cho f n nk, n

Câu 75 Tìm tất hàm số f : *  * thỏa mãn đồng thời điều kiện sau:

i) f  0 0,f 1 1

ii) f 0  f  1  f 2 

iii) f x 2y2 f x2  f y2 ,x y,  *

Câu 76 Tìm tất hàm số f :  thỏa mãm điều kiện sau:

i) Nếu a b f a  f b 

ii) f ab  f a b2 f a  f b ,a b, 

Câu 77 Tồn hay không hàm số f : 1, 2, , n thỏa mãn điều kiện:

i) f hàm đơn ánh

ii) f ab  f a  f b  với a b, 1,2, ,nab n

Câu 78 Giả sử Josephus có n1 người bạn, n người thành vòng tròn

đánh số từ đến n theo chiều kim đồng hồ, tự sát theo nguyên tắc, người thứ cầm dao đếm tự sát, người thứ hai đếm tự sát,<Quá trình dừng lại người Gọi f n  hàm số biểu thị vị trí cùa người sống sót Câu hỏi đặt là, tính

 

f n ?

Câu 79 Cho hai hàm số f g, : *  * hai hàm số thỏa mãn đồng thời điều kiện:

i) g hàm số toàn ánh

ii) 2f n2 n2g n2 , n

Nếu f n  n 2019 n n,   f có vơ số điểm bất động

Câu 80 Tìm tất hàm số g: *  * thỏa mãn điều kiện sau:

 

     

g g nn g n1   3 n g n , n

Câu 81 Cho ba số thực a b c, , không âm, phân biệt cho tồn hàm f g, :   thỏa

mãn af xy  bf x cf x  g y 

y

 

   

(15)

TẠ

P

CH

Í VÀ

LI

ỆU TO

ÁN

H

C

Chứng minh tồn hàm h:   cho:

  x    ,

f xy f f x h y x y

y

 

      

 

Câu 82 Tìm tất hàm số f :  thỏa mãn:

     

! ! ! ! , ,

n f m f n  f mm n

Câu 83 Tồn hay không hàm số f : *  * thỏa mãn điều kiện sau:

 

  3 2  , *

f f nnf n  n

Câu 84 Tìm tất hàm số tăng thực f : *  *thỏa mãn điều kiện sau:

 

  2  , *

f n f n  f n  n

Câu 85 Tìm tất toàn ánh f :  cho với m n,  thỏa mãn:

   

(16)

CH

INH

PH

C

OL

YM

PIC T

O

ÁN

II LỜI GIẢI

Câu Tìm tất hàm số f :  thỏa mãn điều kiện sau:

      

3f n 2f f n n n,

Lời giải

Giả sử f hàm số thỏa mãn điều kiện toán Đặt g n  f n   n n,

Khi đó, ta 2g f n  2f f n   f n  f n  n g n , n 1  Áp dụng liên tiếp  1 ta

 2   22      2m      ,

g n g f n g f f n g f f f nm dấu f

Như g n  chia hết cho ,m   m g n   0, n hay f n   n n, Thử lại thấy hàm số f n   n n, thỏa mãn yêu cầu đề

Vậy tất hàm số thỏa mãn đề là: f n   n n,

Nhận xét Việc đặt hàm phụ g n  f n   n n, giúp ta đưa phương trình hàm ban đầu dạng đẹp Và ta phát thêm tính chất hàm

 

g n để từ ta áp dụng liên tiếp tính chất kết hợp với tính chất số học chia hết để suy hàm thỏa mãn yêu cầu đề

Câu Tìm tất hàm số f :  thỏa mãn điều kiện sau

m n f m   n2mf n nf m ,m n,  1 

Lời giải

Giả sử f hàm số thỏa mãn điều kiện tốn Kí hiệu P u v ,  phép u v, vào  1 ta được:

 0,   2   0 , 

P n nf n nf n

Do f n 2  f  0 , n Đặt g n  f n  f 0 , n

Khi đó, ta thay vơ  1 ta m n g m   2n2mg n ng m ,m n,  2  Hơn nữa, ta cịn có g 0 0 g n 2   0, n

Kí hiệu Q u v ,  phép m u ,nv vào  2

 , 2  2 2  , 

Q n n ng n ng n n

(17)

TẠ

P

CH

Í VÀ

LI

ỆU TO

ÁN

H

C

2 ,2 23  5   , 

Q n n n g n n g n n

Từ suy rag n 3 5g n 4 , n

Từ ta áp dụng liên tục tính chất trên, ta ta suy   3 ,k   ,  *

g n n k

Suy ra: g n   0, n hay f n  f  0 const n,  Thử lại ta thấy hàm thỏa mãn yêu cầu toán

Vậy tất hàm số thỏa mãn yêu cầu toán f n  f  0 const, n

Nhận xét Cũng tương tự bài tốn 1 ta nhìn phương trình hàm ban đầu hàm phụ khác, phép ta phát số tính chất sơ khai ban đầu Và phép đặt g n  f n   n n, ta phương trình hàm có dạng y chang phương trình hàm ban đầu, ta lại thêm điều kiện ràng buộc g 0 0

 2   0,

g n n nên từ ta thêm ràng buộc, thuận lợi cho việc giải phương trình Phép đặt hay, vừa bảo tồn phương trình hàm có dạng y chang ban đầu kèm theo điều kiện rang buộc mà phương trình hàm ban đầu khơng có Từ đấy, tương tự bài tốn 1, ta phát tính chất hàm g n  sử dụng liên tục chúng kết hợp với tính chất chia hết để suy hàm số cần tìm

Câu Cho hàm số f : *  * thỏa mãn điều kiện sau:

 1   ,  *

f n f f n n

Chứng minh f n   n n, *

IMO 1977

Lời giải

Giả sử f hàm số thỏa mãn điều kiện toán

Đặt dminf n n ,  *,theo nguyên lý cực hạn d tồn Gọi m * cho: f m d

Nếu m1 df m  f f m  1 , mâu thuẫn Do f n  đạt giá trị nhỏ nhất n1

Lập luận tương tự ta có f  2 minf n n ,  *,n2 Và lập luận lại trình tương tự ta được:

 1   2   3   

f f f f n

Ta có f 1 1 nên f n   n n, *

(18)

CH

INH

PH

C

OL

YM

PIC T

O

ÁN

Suy f f n  0  f n 1 , mâu thuẫn

Do đó, f n   n n, *,thử lại thấy thỏa mãn yêu cầu toán Vậy tất hàm số thỏa mãn yêu cầu toán là: f n   n n, *

Nhận xét Đây tốn phương trình hàm kì thi Tốn Quốc Tế - IMO năm 1977, tốn phương trình hàm với điều kiện rang buộc dạng bất đẳng thức, lạ Làm ta ý tưởng sử dụng nguyên lý cực hạn để đánh giá để có điều vơ lý suy hàm số thỏa mãn yêu cầu đề

Câu Tìm tất hàm số f : *  * thỏa mãn điều kiện sau:

     

    

2 , , *

x f y f x y x y Lời giải

Giả sử f hàm số thỏa mãn điều kiện toán

Trong   ta x y 1 ta 1f    1 f2  1 f 1 1

Trong   ta x1 ta 1 f y f  2 1     y y y, * y f y , y * 1  Trong   ta y1 ta

         

            

2 1 1, , * 1 1, , * , * 2

x f f x x y x f x x y f x x x

Từ  1  2 ta suy f x   x x, *,thử lại ta thấy thỏa mãn yêu cầu toán Vậy tất hàm số thỏa mãn yêu cầu toán f x   x x, *

Nhận xét Đây tốn phương trình hàm tập rời rạc, mà cho dạng chia hết Bằng phép đơn giản với đánh giá số học khơng q khó khan, ta nhanh chóng đánh giá biên hàm f để từ ta suy hàm số thỏa mãn đề

Câu Tìm tất hàm số f : *  * thỏa mãn điều kiện sau:

     2    

2 , , * *

f m f n m n m n

IMO Shortlist 2004

Lời giải

Giả sử f hàm số thỏa mãn điều kiện toán Trong  * ta m n 1 ta được:

     2    

2 1 1 12 1 4 1 1,

f f f f  1  f  1 1 Trong  * ta m1 ta được:

     2         2  

2 1 12 , , * 1 1 , *

(19)

TẠ

P

CH

Í VÀ

LI

ỆU TO

ÁN

H

C

Trong  * ta n1 ta được:

     2        2  

2 1 1 , * 1 1 , *

f m f m m f m m m

Với p số nguyên tố thì: Trong  * ta m1,n p 1 ta được:

 

 

1 f p p  

 

   

 

  



1

1

f p p

f p p

Trường hợp 1 f p 1p2  f p 1p21 Ta m p 1,n1 vào  * ta được:

     2 2   2   2 2

2 1 1 1 1 1 1 1 1

f p p p p

Mà ta lại có đánh giá sau đây:

p212 1 p1 2 p12 p p2 12 p2p2 p121 ,2

mâu thuẫn Do đó, ta phải xảy trường hợp lại

Trường hợp 1 f p 1 p f p 1 p 1,với p số nguyên tố Hay tồn k cho f k k

Với k số tự nhiên n0 ta có:

              

     2  2   

2 2 1 1 2

k f n k n k f n p f n p n f n f n n

Khi ta chọn k số đủ lớn ta bắt buộc phải có: f n   n n, *,thử lại thỏa Vậy tất hàm số thỏa mãn yêu cầu toán là: f n   n n, *

Nhận xét Cũng tương tự bài toán 4, phương trình hàm tập rời rạc có dạng chia hết Cũng tương tự trên, ta phép đơn giản để phát số tính chất đề Nhưng bài tốn 5 khó bài tốn 4 nhiều, từ tính chất ta tìm được, ta khơng thể chặn khoảng hàm f để suy f n   n n, * tốn Vì mà ta phải xét giá trị hàm số

(20)

CH

INH

PH

C

OL

YM

PIC T

O

ÁN

Câu Tìm tất hàm f :  thỏa mãn tồn số k số nguyên tố p

cho với n k f n p ,    f n  m n f m 1  f n 1

Iran TST 2005

Lời giải

Giả sử f hàm số thỏa mãn điều kiện toán

Giả sử n kp không chia hết cho n1 tồn k cho n1n kp Suy ta f n f n kp    1

Mặt khác ta lại có f n  f n kp   nên f n f n    1 f n 1 f n 1 Với n1 n1n1kpf n f n   1kp 1

Do với n1 ta có: f n    1, Bây ta xét hai trường hợp sau:

Trường hợp f n   2, n k p n1

Xác định n kp không chia hết cho n1 tồn m cho: n1m p m1 Suy f n f m    1 hay f n 1

Ta xác định hàm f sau:  f n   2, n k p n1

f n   1, n k p không ước n1

f i  f i p  , i k

Trường hợp f n   1, n k p n1

Trong trường hợp f n   1, n k giả sử Sa f a 2 khơng tồn 

,

m n S thỏa mãn m1 n

Ta xác định hàm f sau f n    1, , n

Với S tập vô hạn cho không tồn m n S,  thỏa mãn m1n với 1

n f n   2 n S f x;  1,với giá trị x1 lại f  1 số xác định f    2 f 1

Từ ta thử lại đề thấy thỏa mãn nên ta hồn thành tốn

(21)

TẠ

P

CH

Í VÀ

LI

ỆU TO

ÁN

H

C

Câu Cho p số nguyên tố lẻ Tìm tất hàm f :  thỏa mãn đồng thời

điều kiện:

i) f m  f n với m n modp

ii) f mn  f m f n   ,m n, 

USA TST

Lời giải

Giả sử f hàm số thỏa mãn điều kiện tốn

Với k ,thì ta cóf p k  1 f pk  f p f k    1 f k 0 Bây ta xét hai trường hợp sau

Trường hợp f p 0

Dễ thấy f 1 0 f n   0, n ,mâu thuẫn với f p 0 Xét riêng f 1 1

Với xp khong chia hết cho x ta có y cho xy1 mod  p Do ta có f x f y    f xy  f  1  1, x y, 

Suy ra: f n  1 p không chia hết cho n

Mặt khác ta lại có f n 2  f n2 1 với p không chia hết cho n nên f m 1, m số phương mod p p không chia hết cho m

Nếu không tồn i, với p không chia hết cho i cho f i  1 ta có    1,

f n n p không chia hết cho n

Xét i số khơng phương mod p k số khơng phương mod p p khơng chia hết cho k ta suy ik số phương mod p

Mặt khác ta lại có f k  f i f k    f ik  1 Hay

f x 1, x số phương mod p p khơng chia hết cho xf x  1, x số khơng phương mod p

Xét số x0 cho f x 0  1

Bây từ điều kiện ii) ta thay m x n p 0,  ta được:

   0     0

f p f px f p f x hay f p 1

Suy ra:

f x 1, x số phương mod p

f x  1, x số khơng phương mod p

Trường hợp f p 0 suy f n  0, p n

(22)

CH

INH

PH

C

OL

YM

PIC T

O

ÁN

Khả 2. Nếu f 1 0

Giả sử tồn x0 cho f x 0 0 p không chia hết cho x0 Suy f nx 0  0, n

Ta có dãy x0, , ,x0 p1x0 hệ thặng dư đầy đủ mod p Suy f 1 0, điều mâu thuẫn

Vậy ta có f x  0 p x f x 1, với giá trị x lại

Từ kết đây, ta thấy có hàm số thỏa mãn yêu cầu toán:    0,

f n n  

1

if p n f n

if n

 

 p

   1,

f n n   

1 mod

1 mod

if n la mot so chinh phuong p

f n

if n khong la mot so chinh phuong p

Vậy tất hàm thỏa mãn yêu cầu toán

Nhận xét Đây tốn khó, với điều kiện hàm khó xử lý, tốn khó kì thi chọn đội tuyển IMO của Mỹ, việc ứng dụng sâu sắc kiến thức Số Học tổng hợp lời giải, phức tạp Mong bạn đọc suy nghĩ đọc thật kĩ, mong bạn đọc có lời giải khác ngắn gọn hay cho tốn

Câu Tìm số ngun không âm n nhỏ cho tồn hàm số f : 0, khác

hằng số thỏa mãn đồng thời điều kiện:

i) f xy  f x f y   ,x y, 

ii) 2f x 2y2 f x  f y 0,1, , ,n x y, 

Với số n tìm được, tìm tất hàm số thỏa mãn

Lời giải

Với n1 xét hàm f xác định sau:  

1

if p x f x

if x

 

 p,với p số nguyên tố có dạng 4k3

Hiển nhiên hàm số thỏa mãn yêu cầu toán

Giả sử với n0 tồn hàm số f thỏa mãn u cầu tốn Khi ta có:

  2         2 2         2f x y f x f y 0, x y, 2f x y f x f y , x y,

Từ điều kiện i) ta x y 0 ta được:

 0  2 0   0 0

f f f f 0 1

(23)

TẠ

P

CH

Í VÀ

LI

ỆU TO

ÁN

H

C

Ta y0 vào   ta 2f x 2  f x  f 0  f x   1, xf x 2  f x2 , x nên ta suy ra:

            

2

2f x 2f x f x 1, x f x 1, x ,do f x   0, x

Điều lại trái với giả thiết f khác số

Trường hợp 2.f  0 0

Ta y0 vào   ta 2f x 2  f x  f  0  f x , xf x 2  f x2 , x nên ta suy 2f x2 2f x 2  f x , x Suy với x ta phải có f x 0  

2

f x Nếu tồn x0 cho  0 

1.

f x Ta x y x  vào   ta được:

        2         

0 0 0 0

2 2f x 2f f x 2f x x f x f x 2f x

Từ   ta thay x1,y0 ta được: f  1 0 Từ   ta thay x1,y1 ta được: f  2 0 Từ ta thay f  2 0 vào    ta được:

   0, ,

f x x điều lại mâu thuẫn với f khác số

Vậy từ ta khẳng định n1 giá trị nhỏ thỏa mãn yêu cầu toán

Khi ta tìm n1 ta quay lại việc giải tốn đề

Tìm tất hàm số f : 0, khác số thỏa mãn đồng thời điều kiện:

i) f xy  f x f y   ,x y, 

ii) 2f x 2y2 f x  f y  0,1 ,x y, 

Giả sử f hàm số thỏa mãn điều kiện toán Ta dễ dàng chứng minh rằng: f 0 0, f 1 1 Trong i) ta y x ta f x 2  f x2 , x Trong ii) ta y0 ta

 2    2           2f x f x 2f x f x 0,1 f x 0,1 Trong i) ta x y  1 ta

        

2 1 1 1 1 1

f f f

(24)

CH

INH

PH

C

OL

YM

PIC T

O

ÁN

     1  ,       , 

f x f f x x f x f x x

Trường hợp 1. Tồn số nguyên tố p cho f p 0 Giả sử tồn số nguyên tố q p cho f q 0 Trong ii) ta x p y q ,  ta

  2        2 2 2f p q f p f q f p q Do với a b,  ta ln có:

 2 2  2 2  2 2 2 2    2   2

2f a b f p q 2f a b p q 2f ap bq aq bp

Lưu ý a2 b2p2q2ap bq  2 aq bp 2 đẳng thức Brahmagupta –

Fibonacci nổi tiếng, đề cập đến nhiều sách, mong bạn đọc lưu ý

chi tiết để giải tốn

Vì 0 f x  f y 2f x 2y2 nên f aq bp  0 Do  p q, 1 nên tồn a b,  cho aq bp 1 Suy 1 f 1  f aq bp  0,điều vô lý Vậy tồn số nguyên tố p cho f p 0

Khả 1. Nếu p số ngun tố có dạng 4k1,k tồn a cho p a2 1 hay f a 10

Lưu ý rằng. Kết bạn tham khảo phần chuyên đề Thặng dư bình phương

Mặt khác, ii) ta x1,y a ta được:

 2 2              2  1f a f f a 2f a f f a f a 1, Điều mẫu thuẫn

Vậy từ xảy khả lại

Khả Nếu p số nguyên tố có dạng 4k3

Từ ta cóf x  0 p x f x 1 với giá trị x lại

Trường hợp f p 1 với số nguyên tố p Khi f x   1, x \ 0 

Vậy từ có hai hàm số thỏa mãn yêu cầu toán là:   

1

if p x f x

if x

 

 p, p số ngun tố có dạng 4k3,k

    

 

0

if x f x

if x

(25)

TẠ

P

CH

Í VÀ

LI

ỆU TO

ÁN

H

C

thục Sử dụng nhiều mạng kiến thức liên quan đến số nguyên tố, thặng dư bình phương hay đẳng thức tiếng Toán học Thực hàm liên quan đến số học tổng hợp, hay thú vị, mong bạn đọc nghiên cứu thật kĩ cẩn thận toán

Câu Giả sử hàm số f : *  thỏa mãn điều kiện sau:

 1 1

f  

      

 

  

 

 

    

  

1

1

2

1 2

n

f if n m

f n

n

f if n m

Tìm giá trị n cho f n 2019

Lời giải

Từ cách xác định hàm f ta dễ dàng tính được:

 2   3 2;  4   5   6   7 3

f f f f f f

Bây ta viết dạng nhị phân sau:

           

           

     

     

2 2

2 2

1 1; 10 2; 11

4 100 3; 101 3; 110 3;

f f f f f f

f f f f f f

Từ cách việt dạng nhị phân trên, ta dự đoán f n  số chữ số biễu diễn nhị phân số n

Ta chứng minh dự đoán quy nạp sau Thật vậy, ta thấy khẳng định với n1,n2

Giả sử khẳng định đến n Ta chứng minh khẳng định đến n1 Nếu n số chẵn n a ak k1 0a1  f n  k

Khi ta có n 1 a ak k1 1a1

Từ ta có     

   

         

   

1 1,

2 k k 2

n a a a f n k f n f n k

tức số chữ số biểu diễn nhị phân số n

Nếu n số lẻ cách làm tương tự ta kết tương tự

Vậy theo nguyên lý quy nạp ta suy f n  số chữ số biểu diễn nhị phân n Ta ta suy f n 2019 biểu diễn n hệ nhị phân chứa 2019 chữ số

Vậy từ ta suy ra: 22018 n 22019

(26)

CH

INH

PH

C

OL

YM

PIC T

O

ÁN

nắm quy luật dãy số mà tác giá ẩn toán, từ ta nảy ý tưởng giải việc triển khai, cuối ta thu kết tốn

Câu 10 Tìm tất hàm số f : *  * thỏa mãn điều kiện sau:

       

     

     

 

 

 

    

    

1 1, 3

4 2 3 2

f f

f n f n

f n f n f n

f n f n f n

với số nguyên dương n

IMO 1988

Lời giải

Giả sử f hàm số thỏa mãn điều kiện tốn

Một số ngun dương k có bốn dạng sau: 4 , 4 1, 4 2, 4 3; , 

k n k n k n k n k n

Do đó, từ giả thiết toán, hàm số f xác định cách Ta sử dụng biểu diễn số để tìm biểu diễn hàm số f

Ta có nhận xét sơ sau:

       

       

     

     

2 2

2 2

1 1 , 10 01

11 3 11 , 100 001 ,

f f f f

f f f f

Từ ta thấy quy luật sau:

Quy luật. Biểu diễn f n  hệ số cách viết ngược lại biểu diễn n hệ số tức f a a k k1 a a1 02a a a a0 1 k1 k2

Bây ta chứng minh dự đoán quy nạp sau

Chứng minh

Với n1,2,3, hiển nhiên đúng, ta thử kiểm tra

Giả sử tính chất cho với k n Ta chứng minh tinh chất với n

Trường hợp 1. Nếu n2m theo giả thiết ta có f m  f n  Vì n2m nên m biễu diễn hệ số dạng ma ak k1 a a1 2 na ak k1 a a1 00 2

Mà theo giả thiết quy nạp ta có:

 

k k1 00 2      k k1 02 k1 k 2  0 k1 k2

f a a a a f n f m f a a a a a a a a a a a a

Từ đây, trường hợp này, tính chất chứng minh

Trường hợp Nếu n4m1 với ma ak k1 a a1 2 na ak k1 a a1 0012

  

  1 1 0 2

2m a ak k a a

(27)

TẠ P CH Í VÀ TƯ LI ỆU TO ÁN H Ọ C                                                                 

1 2

1 1

0 1 2 1 2

0 1 1

0 1 1

2 2

0 1

2

01 2

2

2

10 10 10

k k

k k k k

k k k k

k k k k

k k k k

k k

f a a a a f n f m f m f m

f a a a a f a a a a

a a a a a a a a

a a a a a a a a

a a a a a a a a

a a a aa0 1a a a k1 k2

Từ đây, trường hợp này, tính chất chứng minh

Trường hợp 3. Nếu n4m3 với ma ak k1 a a1 2 na ak k1 a a1 0112

  

  1 1 0 2

2m a ak k a a

Mà theo giả thiết quy nạp ta có:

                                                    

1

0 1 1 1

0 1 2

0 1 2

11 3 2

2 2

1

1 10 11

k k

k k k k k k

k k k k

f a a a a f n f m f m f m

f m f m f m

a a a a a a a a a a a a

a a a a a a a a

Từ đây, trường hợp này, tính chất chứng minh

Vậy theo nguyên lý quy nạp quy luật chứng minh Vậy tất hàm f n  thỏa mãn đề là:

 

k k1 2 k1 k2,

f a a a a a a a a

trong na ak k1 a a1 2 biễu diễn số n hệ số

(28)

CH

INH

PH

C

OL

YM

PIC T

O

ÁN

Câu 11 Cho hàm số f :    thỏa mãn đồng thời điều kiện:

f n  ước n2018 với n

f a f b     f c  với a b c, ,   a2b2 c2

a) Chứng minh n lẻ n f n 1

b) Gọi A tập hợp giá trị có f 2  f2018 Tính A

Lời giải

a) Nếu n1 mod 2  ta chọn              

     

2018

2 1 1 1 1

, , , , | |

2 2

n n a a

a b c n f n f

Ta có                 

      

 

2018

2

2018 2018 2018

| |gcd , gcd , 1

2

n n

f n n f n n n f n

Nếu n0 mod 4  ta chọn a b c, , 4 , 4k k2 1, 4k2 1 f 4 | 4k fk2 1 | 4  k212018 Ta có f    4 | 4k k 2018 f  4 |gcd 4k  k 2018, 4 k2120181

b)Theo câu a, ta có f 2 22018 f 2 2 , , , 20 2018

Với a b c, , 2018,1018080,100921 ta có        

   

  

2018

2

2018

2018 1009 1009 2018 2018

f f

f

       

f 2018 gcd2018 2018,10092 1 22018 f 2018  2 , , , 20 2018

Với a b c, ,  thoả  

 

  

 

 

2 2 *

gcd , ,

a b c

a b c

 Nếu a b, lẻ,c chẵn

Do a2b2 c2      2a  2b  2c nên 1 f  2cf    2a f 2bf :   nên f  2af  2bf  2c 1

 Nếu  

 

   

 

1 mod mod

a c

b

Khi đó: f 2cf   2a f 2bf 2 ,aa c, 1 lẻ thỏa mãn (*) Do f  2cf  2a * *   b :a2 b2 c2,a c, 1 Ta có f2018 thỏa  * * với     

 

2

2018 1018080 1009

, , , ,

2 2

a b cf 2 không thỏa

 * * a c, 1 nên f 2  f 2018

Dễ dàng chứng minh 2a2b 2c2d với    a b,  c d, cách chia hai vế cho

(29)

TẠ

P

CH

Í VÀ

LI

ỆU TO

ÁN

H

C

Câu 12 Có tồn hàm số f S: Sthỏa mãn điều kiện

     2 , ,  , 

f a f b f a b a b S a b khơng, S *\ 1 ?

Lời giải

Giả sử tồn hàm số thỏa mãn u cầu tốn Ta tìm tính chất đặc biệt hàm số f Xét hai biểu thức sau f a f d f c f a f b f c           ,

Ta có f a f d f c      f a d f c 2   f a d c 4 2

          2  4

f a f b f c f a f b c f a b c Ta chọn c d, cho a d c4 a b c2 4 a d2 b c4 ad2 b c2  1 Với c d, thỏa mãn (1)

 4 2  4                  2

f a d c f a b c f a f d f c f a f b f c f b f d

Tất nhiên ta cần chọn d bc thỏa mãn  1 chẳng hạn d b c ab 2,  Nên từ  2 ta f b  f b 2 , b S (3)

Từ tính chất  3 ta có

     2 , ,  ,        , ,  ,   4

f a f b f a b a b S a b f a f b f ab a b S a b

Sử dụng tính chất    3 , ta f 16  f  42  f  4  f  22  f  2 Mặt khác f 16  f 2.8  f   2 f

Từ hai đẳng thức ta f   2 ff 2  f  8 1 (vơ lí.) Vậy khơng tồn hàm số f thỏa mãn yêu cầu tốn

Câu 13 Tìm tất hàm số f : *  * thỏa mãn điều kiện

  2       2   *

1 ,

n f n f f n n n n

Lời giải

Với dạng toán mà giả thiết bất đẳng thức việc dự đốn nghiệm đóng vai trò quan trọng việc định hướng lời giải Việc dự đốn nghiệm thường ta thơng qua việc tính f      1 ,f ,f ,

Thay n1 vào phương trình ban đầu ta 0 f 1 f f  1  2 f  1 1 Thay n2 vào phương trình ban đầu ta 1 f 2 f f  2 6

Từ bất đẳng thức suy f 2 1, f 2 3 Từ điều kiện ban đầu ta thay n f n  thu

 

f n 12  f f n f f f n       f n 2 f n , n *

(30)

CH

INH

PH

C

OL

YM

PIC T

O

ÁN

 

           

 4 f 1  f f f f ff f 2

Điều mâu thuẫn với 1 f 2 f f  2 6 Do f 2 2

Do ta dự đoán f n   n n, * Ta chứng minh phương pháp quy nạp Giả sử f 1 1,f  2 2, ,f n   1 n Ta chứng minh f n n

Với số nguyên dương n, đặt f n m Ta xét trường hợp sau:  Nếu m n  1 f n f f n    mf m m2n12 vơ lí

 Nếu f n    m nn1f f n   f n f f n    n2nf f n  n  

 

       .       2 

f f f n f f n f f n f f f n f f n n ,

Kết hợp với f f f n   f f n  f n 12 ta đượcn2 f n 12  f n  n vơ lí Do f n n Vậy f n   n n, *

Câu 14 Tìm tất hàm số f :  thỏa mãn đồng thời hai điều kiện sau:

i) x f y f x     y f x f y    với x y,  ;

ii) Tập hợp        

 

 , , , 

f x f y

I x y x y

x y khoảng

Lời giải

Phân tích Điểm mấu chốt để giải tốn hiểu tận dụng giả thiết ii) Tập hợp I khoảng có nghĩa a b I a b,  ,   a b, I

Từ điều ta suy tính chất quan trọng tập hợp I có số thực a Ix a x I  , x a x I  ,

Khi ta có lời giải sau

Để ý f x  f y  x y x y ( i) ) Do 1I

Giả sử t I Khi t1 tồn x y,  ,x y cho f x f y  t x y  Vì x y nên f y f x    f x f y   

 

    

 

      

   

 

   

  

1

f y f x f x f y y x

y x t x y t

y f x x f y

Do 

1 t II khoảng nên phần từ x nằm t

1 t thuộc I

Nếu t1  

0

1 t (mâu thuẫn ta lại có 1I)

Do t1 Tương tự ta có  

1 1

(31)

TẠ

P

CH

Í VÀ

LI

ỆU TO

ÁN

H

C

Như ta chứng minh t I  

1 t I t0

t0  

1 0

1 t nên ta có mâu thuẫn

Như I phải tập rỗng mà rõ ràng I , chẳng hạn f   1 f I Do không tồn hàm số thỏa mãn yêu cầu tốn

Ngồi ta giải toán sau

Đặt        , f x f y

F x y

x y với x y,  ,x y

Tương tự cách đầu, ta có F x y ,  1, x, y ,x y

Nếu tồn x y cho F x y , 0 F y f x x f x   ,   1 (vô lý) Do F x y ,  0;1 ,x y,  ,x y  *

Suy f đơn ánh Do với x y,  ,x y ta có

 

               

f x f y f y f y f y f x f y f y x y

Hay F x f y y f y   ,   F y f x y f y F x y   ,     , 1

Từ đây, kết hợp với  * ii) ta suy F x y ,  1, x y,  ,x y Suy f đơn điệu tăng

 Nếu x yf x  f y  x y hay y f x   x f y  Suy f y f x    f x f y   , kết hợp với i), ta x y  Tương tự x y ta có mâu thuẫn

Vậy khơng tồn hàm số thỏa mãn yêu cầu toán

Câu 15 Tìm hàm số f : *  * thỏa mãn f m2  f n m  n2,m n,  *

Lời giải

Giả sử tồn hàm f thỏa mãn yêu cầu tốn Ta có f2 1  f  1 4 f 1 1

Thay m  1 f n  1n2, n

Thay m1,n p 1p P     

 

   

    

   

2

2

1

1

1

f p p

f p p

f p p

Nếu  p P f p:   1 p21

(32)

CH

INH

PH

C

OL

YM

PIC T

O

ÁN

p2  1 p2 2p2 nên có mâu thuẫn Vậy f p     1 p 1, p P

Thay m p  1 p12 f n p12n2 Suy             

2

2

1 mod

n p p f n

Mà f n n2 0 mod p12 f n  Suy f n n2 0 mod p12 f n 

Nếu f n n, ta có f n n2  f n( )p1 ,2  p P * Mà tập số nguyên tố vô hạn nên f n p12   Mà f n n2 cố định với n xác định

Do cần chọn p nguyên tố đủ lớn ta có mâu thuẫn  * Vậy f n   n n, * (thử lại thỏa mãn)

Câu 16 Cho hàm f x y ,  thỏa mãn điều kiện:

f 0,y  y 1;f x 1,0 f x ,1  f x 1,y1 f x f x ,  1,y

Với số nguyên không âm x y, Tìm f4,1981

Lời giải

Ta có f  1,nf 0,f 1,n1 1 f1,n1 Do f  1,n  n f  1,0  n f  0,1  n Ta lại có f2,n f 1,f 2,n1 f2,n 1 Do f 2,n2n f 2,02n f  1,1 2n3 Bây f 3,n f2,f 3,n12f 3,n 1 Đặt un2un1 u0  f  3,0  3 f  2,1  3

Do un 2n3f 3,n2n33

Ta có

      

     

 

    

    

  



4,

224

4, 3, 4,

4,0 3,1 13 4, 2

f n

f n f f n

f f

f

Bằng qui nạp ta chứng minh f 4,n222 243

(33)

TẠ

P

CH

Í VÀ

LI

ỆU TO

ÁN

H

C

Câu 17 Cho hàm f :   thỏa mãn điều kiện sau:

iii) f n 1 f n ; n

iv) f f n  3 ,n n Z  

Hãy tính f2003 

Lời giải

Từ    i , iif 1  f f  1  3 f 1 2 Ta có f 2  f f  1 3.1 3

 3    3 3.2

f f f

 2.3    3 3.3 3

f f f

Suy f 2.3n 3 ,n1  n ;f 3n 2.3 ;n  n Z

Nên có f  3n1  f f  2.3n 2.3n1 f 2.3n1 f f  3n1 3.3n1 3n2 Do khẳng định với n

Ta có 3n1 số nguyên m nằm 3n 2.3n giả thiết  i f n 1 f n  nên có 3n1

số nguyên m nằm f 3n f  2.3n suy 0 m 3nf 3nm2.3n3n Do giả thiết  ii suy f2.3nm f f 3nm3 3 nm

Vậy f2.3nm 3 3nm với 0 m 3n

Suy n2003 2.3 6545 f20033 3 65453822

Câu 18 Cho f n  hàm số xác định với n * lấy giá tị khơng âm thỏa mãn

tính chất:

 n m,  *: f m n   f m f n 

lấy giá trị  f 2 0 f  3 0

f99993333 Tính f 2000

Lời giải

f m n   f m  f n  lấy giá trị nên ta suy f m n   f m  f n 

       

f 2 1ff  0 f 1

Ta có f  6  f 3  f  3 2  9   6   3 3

f f f

9999 9996  3 3333

(34)

CH

INH

PH

C

OL

YM

PIC T

O

ÁN

Vì giả thiết cho f 99993333 nên ta có dấu “=” bất đẳng thức xảy ra, tức  3   , 1, 2, , 3333

f n n nf1998666,f 2001667

Mặt khác aa b,  * a b  f a  f b  f a b   f b 

   

666 f 2000 667 f 2000 666 667 Giả sửf 2000 667  f40001334 f 60001334 667 2001  Mà f 60002000, mâu thuẫn Vậyf 2000666

Câu 19 Cho f g, hàm xác định thỏa mãn điều kiện

     2    , , 

f x y f x y f x g y x y

Chứng minh f x 0 f x    1, x g y 0  a

Lời giải

Ta dùng phương pháp phản chứng Giả sử lại điểmy0 : g y 0  a

Ta lấy x f x0 :  0 0 xây dựng dãy x kk 0,1, như sau:

   

   

    

  

   



0 0

1

0 0

,khi ,khi

k k k

k

k k k

x y f x y f x y

x

x y f x y f x y

Theo giả thiết ta có f x k1  f xky0  f xky0  f xky0  f xky0

     

2 f xk g y0 2a f xk

Nên f x k1 a f x k với a1;k1, 2,

Do ta có:     0

k k

f x a f x

Nhưng f x 0 0 a1 nên chọn k cho a f xk  0 1 dó f x k 1

Mâu thuẫn với giả thiết Vậy g y    1, y R

Câu 20 Cho hàm số f :  thỏa điều kiện

iii) f x    1 x x;

iv) f x y   f x f y   ;x y, 

Chứng minh tồn hai số a b;  mà f a f b    0

Lời giải

Ta chứng minh f x   0, x

(35)

TẠ P CH Í VÀ TƯ LI ỆU TO ÁN H Ọ C

Với x 1, trước hết ta chứng minh bất đẳng thức:

         

 

 

2

, ,

2

n

n

x

f x f x n

Với n0 bất đẳng thức đúng!

Giả sử  1 với n k 0 tức      

    2 k k x f x f

Ta có                                       

2 2

1 1

2 2 2

k k k

k k k k k

x x x x x

f f f f tức  1 với n k 1

Theo nguyên lý quy nạp toán học bất đẳng thức  1

Bây chọn n đủ lớn để x 2 ,n x tùy ý,    

 

1

2n 2n

x x

f

Do    

    2 n n x

f tức f x   0, x

Như tồn hai số a b;  mà f a f b    0

Câu 21 Cho  , y 2003cos 2   cos   

2

f x x y a x y với a, 

Chứng minh minf x y , 2 maxf x y , 2 2003

Lời giải

Ta có    

 

2003

0,0 ;

2 2

f f

Nên           

 

2003

max , max 0,0 , , ,

2 2

f x y f f x y max  , 2 2003

f x y Ta lại có          

   

2003 2003

; sin , ; sin

4 4

f a f a

Nên     

   

2003

; ;

4 4

f f

Suy            

   

 

2003

min , ; , ; ,

4 4

f x y f f x y

 

 

 

 ,  2003

2

f x y

(36)

CH INH PH Ụ C OL YM PIC T O ÁN

Câu 22 Cho hàm số     

2 1

,

x

f x x

x

Giả sử f x0 x f xn   f fn1 x     n *, x

Chứng minh  

 

1

1

, 1,0,1

1 n n n f x n x

f x x

f x                 Lời giải

Đặt   1 12  12 

n n

n

p x x x   1 1 2  12  , ,  

n n

n

q x x x x y

Ta có:                                       2 1 0

2 , ,

,

n n n

n n n

p x p x q x

q x p x q x x y

p x x

f x x x

q x

Giả sử        k k k p x f x

q x  

                                  2 1 1 2 k

k k k k

k

k k k k

k

p x

q x p x q x p x

f x

p x p x q x q x

q x

Do      

n ,    ,

n

n

p x

f x n x

q x

Ta có      n , x 1,0,1 có:                                                               1

2 2

2 2

1

1 1

1 1

n n n n

n n n n

n n

x x x x

f x

f x x x x x

                                                  1 2 2

2 2 2

1 2 1 1 1

1

1 1 1

1

n n

n n

n n n n n

x x x x

x x x x f x

x

Câu 23 Cho hàm số f : * *  * hàm số thỏa mãn đồng thời điều kiện sau:

i) f  1,1 2

ii) f m 1,n f m n , m m n, ,  *

iii) f m n , 1 f m n ,  n m n, ,  *

Tìm tất cặp số  p q, cho f p q , 2019

Doãn Quang Tiến

(37)

TẠ

P

CH

Í VÀ

LI

ỆU TO

ÁN

H

C

 ,   1,    1  2,    2    1   1,  1

p p f p q f p q p f p q p p f q

              

 1,    1 1   1,1   2019,

2 2

p p q q p p

f q q f áp dụng điều kiện

iii)

Từ điều kiện i) ta có f 1,1 4 nên từ đẳng thức ta suy ra:

         

      

   

     

 

       

1 1

1,1 2019

2 2

1

2017 2.2017

2

q q p p q q p p

f

p p q q

p q p q

Vì 2017 số nguyên tố p q   p q 1 nên xảy trường hợp sau:

Trường hợp 1. p q 1 p q  1 4034 từ ta được: p2018,q2017

Trường hợp 2. p q 2 p q  1 2017 từ ta được: p1010,q1008

Vậy tất cặp số  p q, cho  p q, 2019 là:   p q,  2018, 2017 , 1010,1008   

Câu 24 Tìm tất hàm số f :  thỏa mãn điều kiện sau:

i) 0 f x x2, n

ii) f x  f y  chia hết cho x y với x y,  ,x y

Lời giải

Ta phải xét hai trường hợp

Trường hợp 1. f hàm số

Giả sử f x  c const,c số thuộc

Cho x0 từ i) ta suy ra: 0 f 0  0 f  0 0 Vậy f x   0, x

Trường hợp 2. f không hàm số

f  0 0 nên cho y0 từ điều kiện ii) ta được: x f x , x \   Với x1 từ điều kiện i) ta suy ra: f  1 0 f 1 1

Khả 1. f 1 0

Cho y1 từ điều kiện ii) ta suy ra: x1  f x , x \   Mà ta có x f x , x \ 0  x x, 11

Nên x x 1  f xx x 1x2 x f x  Mặt khác ta có  

 

 

  

2

f x x

x f x

   

  

 

 

2

f x x x

f x x

(38)

CH

INH

PH

C

OL

YM

PIC T

O

ÁN

Vậy khả 1 có hàm số f x x2   x x, thỏa mãn yêu cầu toán

Khả 2. f 1 1

Khi x f x , x \ 0  x1  f x , x \ 1  ta lấy k tùy ý chof k mk, với m tùy ý

Từ điều kiện ii) có k1 mk 1 k1 mk 1 mk m   mk1 m1 Mà từ điều kiện i) ta f k k2 mk k 2 m k

Mặt khác ta lại có    

 

  

      

 



1

1

1

k m

k m k m

m

Do m 1 nên suy ra: m1 m k

Vậy từ ta f x x2, x f x   x x, Thử lại thấy hàm số thỏa mãn yêu cầu toán Vậy tất hàm số thỏa mãn đề

 0,   ,   2,   2  , .

f x f x x f x x f x x x x

Câu 25 Tìm tất hàm số f : *  * mà tập * xx0 thỏa mãn:

   2     , ,  * 1 

f xy

f x f y xyf xy x y

f x y

Lời giải

Ta giải tốn thơng qua ba bước sau

Bước 1. Ta chứng minh f 1 1

Thật vậy, cho y1 vào  1 đặt f  1 a ta

                  

  

2

1

f x f x

f x a xf x f x

f x x f x a

Từ ta suy          

  

1

1 4 1

2 , 5 , *

4

4

a

f f f

a a a a a

Mặt khác, ta cho x y 2 vào  1 ta        

  

2

4

f f f

f

Mà từ  * ta suy       

 

1

2 1 1

4 5a 4a a f

Từ ta xong bước

Bước Ta chứng minh     

   

    

 

2 2 1, 1,

f x

f x n n

(39)

TẠ P CH Í VÀ TƯ LI ỆU TO ÁN H Ọ C

Từ  2 suy  3 n1 Giả sử đến n k

Thì ta có    

             

1

f x k f x k

x k f x k

                                               2 2

1 2 1

1

2

f x

k kx f x f x

x k f x k k x f x k kx f x

Từ theo nguyên lý quy nạp ta suy điều phải chứng minh Vậy khẳng định bước 2 chứng minh

Vậy từ ta suy     

   

  

  

2

1

1

2 1

f f n

n n f n hay     

1 , 1,

f n n

n

Bước Ta chứng minh:          

   

   

2

2

1 , 1, 4

1

f n n

n

n

Thật vậy,  3 ta thay x

n  

                     1 f n f n

n n f n

Tiếp theo, ta thay y

x vào  1  

              

1 2

1

f x f

x f x

x

Vậy         

   

  

   

   

2

1 2 2

1

f n f n

n f n f

n n

Mà ta có: f n  12

n nên suy ra:     

2

1 .

f n

n

Nên từ bước 3 chứng minh hoàn toàn

Thử lại thấy hàm số thỏa mãn yêu cầu toán

(40)

CH

INH

PH

C

OL

YM

PIC T

O

ÁN

Câu 26 Cho hàm f :   hàm số thỏa mãn với n1 có số

nguyên tố p ước n cho:           

n

f n f p f

p

  32018  52019  720202017.

f f f

Hãy tính giá trị biểu thức G f 20182018  f 20192019  f 20202020

Doãn Quang Tiến

Lời giải

Thay n p vào  1 ta    1       1

f f p f f p f p

Thay n pn ta f p   nf pn1 f p  Bằng phương pháp quy nạp ta chứng minh

        

 

2 1 *

2

n n

f p f

Thật vậy,  * với n1 Giả sử  * với n k 1

Ta chứng minh  * với n k 1 hay ta có

                    

 

   

1 1 1

2 2

k k k f k

f p f p f p f f

Từ ta suy  * trường hợp n k 1 Vậy theo nguyên lý quy nạp  * với n Khi ta suy

     

     

   

  

  

     

      

     

    

2018 2019 2020

3 2017

2 2018 1 2019 1 2020 1 2017

2 2

6051 1 2017 1 2.

2

f f f

f f f

f f

Từ  1  2

f    1

f

f p suy ra:   1.'

f p Khi   1

3

f p  1 viết lại sau           

1

,

n

f n f n

p

Cho k số thừa số nguyên tố n, số lượng thừa số nguyên tố     

n

p

k1 

(41)

TẠ

P

CH

Í VÀ

LI

ỆU TO

ÁN

H

C

 Với k2 ta chọn n10 2.5 theo  2

ta được:

            

 

10 1 1 2

10

2 3 3

f f f

 Với k3 ta chọn n12 2.3.4 theo  2 ta được:

                      

   

12 1 1 1 1 1

12

2 3 3 3 3 3 3

f f f f f

Từ đó, phương pháp quy nạp ta suy được:   2,

3

k

f n với k số thừa số nguyên tố n * *  Mà lưu ý 2018 2.1009, 2019 3.673, 2020 5.101.  

Do suy ra: 20182018 có 4036 thừa số nguyên tố, 20192019 có 4038 thừa số nguyên tố,

2020

2020 có 8080 thừa số nguyên tố

Từ theo công thức  * * ta suy được:

        

 20182018  20192019  20202020  4036 4038 8080 16148   .

3 3

G f f f

Vậy từ ta có kết tốn

Nhận xét Một điều thú vị ta thay 2017,2018,2019,2020 x1, ,x x1,x2 ta kết  1  2

3

f không thay đổi

Câu 27 Tìm tất hàm số f : *  * thỏa mãn:

            

3 2 2 *

2f m n f m f n f m f n , m n,

Lời giải

Giả sử tồn hàm f thỏa mãn yêu cầu toán

Trường hợp 1. Nếu f hàm số

Tức f n const c , với c số hiển nhiên thỏa mãn yêu cầu tốn

Trường hợp 2. Nếu f khơng hàm số

Nếu tồn m n,  * cho f m  f n  ta gọi a b, hai số thỏa mãn:     min     , ,  * 1 

f a f b f m f n m n

Giả sử f a  f b  ta có 2f b3  f a f b2    f a f b   2 2f a3  Suy f b  f a b2 f a  f a 2b2 f b  f a f b 

(42)

CH

INH

PH

C

OL

YM

PIC T

O

ÁN

Do có f hàm số thỏa mãn yêu cầu toán

Vậy tất hàm số thỏa mãn đề là: f n constc, với c số

Câu 28 Giả sử f :  hàm liên tục giảm cho với x y,   ta có

           

f x y f f x f y f y f x

Chứng minh f f x  x

Lời giải

Cho y x ta được: f  2xf 2f x  f2f x f x   

Thay x f x ta có f 2f x  f2f f x   f2f f x   f f x   Trừ hai phương trình ta suy ra:

 

 

2   2  2       2    

f f f x f x f f f x f f x f f x f x

Nếu f f x  x, vế trái phương trình âm, đó:

    

      

f f x f f x f x f x f x  f f x   x f x 

là điều mâu thuẫn

Tương tự, ta có điều mâu thuẫn xảy f f x  x

Vậy f f x  x, điều phải chứng minh

Câu 29 Cho song ánh f :  Chứng minh tồn vô số a b c, ,  với

 , ,

a b c thỏa mãn a b c  2f b  f a  f c 

Lời giải

Ta xây dựng dãy  an sau:

Trong số từ 0, 1, 2, , m chọn số a1 cho f a 1  f i   i 0;a m1  

Chọn a2 a1sao cho f a 2  f i , i 0;a2

Chọn akak1 cho f a kf i , i 0;ak

Vậy ta có dãy a1 a2   akak1 f a   1  f a2   f a   kf ak1 Trong ai   f a if j  j 0;ai

Vì f song ánh nên f a   k1  f akp p N,  * Và  c để f c  f a k1  p f a k1

Mặt khác

   

 

   

 

  



1

1

, 1, , 1,

k i

k n

a a i k

(43)

TẠ

P

CH

Í VÀ

LI

ỆU TO

ÁN

H

C

Nên c ak1

   

         

 

  

   

 



1

1

2

k k

k k

k

p a f a p

f a f a f c

f c f a p

Do cách xây dựng, dãy  an dãy vô hạn nên tồn vô số a b c, , thỏa điều kiện nêu

Câu 30 Có hàm f : * * thoả mãn đồng thời điều kiện sau

a) f 1 1

b) f n f n   29f n 121997,  n *

Lời giải

Gọi D tập hợp tất hàm số f thoả mãn điều kiện toán Theo giả thiết b) ta có

f n f n   2f n 121997;

f n 1 f n3f n 22 1997

Suy f n f n   2f n 12  f n 1 ( f n3f n 22 1997

    

     

    

  

 

2

, *

1

f n f n f n f n

n

f n f n

Vì ta có    

          

   

   

1

2

f f f f f n f n

f f f n

Đặt    

  

2

f f c

f  1 suy f n 2cf n  1 f n , n * 2 

Ta chứng minh c * Thật vậy, cp

q với p q,   p q, 1 từ  2 ta có

   

  2   1 ,   *

q f n f n pf n n

Suy q f n 1 ,  n * hay q f n f n2    2 ,  n * n2 Vì 1997f n f n   2f n 12 q2

Mà 1997 số nguyên tố nên q2 1 hay q1 suy c *

Gọif  2 a,do  1 ta có ac  1 f 3 suy ac 1 f  3  f   1 f f  2 21997

ac 1 a2 1997  a c a  1998

Ta a1998, hay f  2 ước dương 1998

Ngược lại với ước dương a 1998 ta xây dựng hàm f : * * sau  1 1;

f f 2 a

 2      1  ,  *

f n a b f n f n n ; b1998 *

(44)

CH

INH

PH

C

OL

YM

PIC T

O

ÁN

Ta chứng minhf thoả mãn điều kiện đề bài, nghĩa f D Thật

                             

          

      

           

       

       

     

2

2

2

1 2

1 2

2

2 , *

f n f n f n f n a b f n f n f n a b f n f n f n f n

f n a b f n f n f n f n f n f n n

Suy f n 1 f n3f n 22  f n 2  f n f n 12

           

f ff 2  ff 2

Từ ta có f n f n   2f n 12  f 3 f 2 2           

a b f 2  f 1  f 2  a b a  1 a2

ab 1 1998 1997. 

Vậy ta f n f n   1f n 121997 hay f D

Ta có tương ứng, mỗif D với giá trị f  2 1998 song ánh D tập ước dương 1998

Do số phần tử D là: D d 1998d2.3 373 1 1 1     16 Vì có tất 16 hàm số thoả mãn đề

Câu 31 Tìm tất hàm số f : *  * cho

a) f 2 2

b) f m n  f m f n    với  *

,

m n , UCLN m n , 1 c) f m    f n   * 

, ,

m n m n

Lời giải

Chọn n1 , thay vào f m n  f m f n     f  1 1

Ta để ý f                   3 ff 15  f ff f 10  f f f Vậy f     3  f f 4 Mà 2 f   2  f 4 nên f 3 3

Từ ta tính f 4 4,f 5 5, f 6 6, f 7 7, f 8 8, f 9 9, f  10 10 Dự đoán f n n với  n *

Giả sử f k k với  k *,10 k n Ta chứng minh điều khẳng định với

 1

k n

Nếu k số chẵn, ta xét hai trường hợp sau:  k2 2 l1 ; ,  l *

(45)

TẠ

P

CH

Í VÀ

LI

ỆU TO

ÁN

H

C

Lúc f k  f22l1   f 2 f2l 1 22l 1 kk2 ;   *

Lúc f k 2 f22 f2 2 11 f 2 f21 1 2 21  1 k Mặt khác k 1 f k  1   f kf k 1 f k2 k

Do f k k f k,    1 k

Nếu k số lẻ k1 số chẵn, ta xét hai trường hợp sau:  k 1 22l1 ; ,  l * Khi 0 2  n,0 2  l 1 n

Theo giả thiết quy nạp f k  1 f22l1   f 2 f2l 1 22l  1 kk 1 f k  1   f kf k   1 k f k k

     *

1 ;

k Lúc

 

         1     1   1  

1 2 2 2 2

f k f f f f k

Mặt khác k 1 f k  1   f kf k 1 f k2  f k3 k Do f k k f k,    1 k 1,f k 2 k

Theo nguyên lí quy nạp f n n với   *

n

Câu 32 Tìm tất hàm số f :  thỏa mãn

         1, , 

f m n f mn f m f n m n  1

Lời giải

Thay m n 0 vào  1 ta 2f 0  f2 0  1 f 0 1 Thay m1,n 1 vào  1 ta lại có f  1 f   1 f  1 Vậy f   1 f  1 1

 Xét f 1 1 Thay n1 vào  1 ta có : f m  1    f mf m   1, m

Suy f m(    1) 1, m hay f m( ) 1,  m Thử lại thỏa mãn  Xét f  1

Thay n 1 vào  1 ta f m  1  fm   1, m  2

Thay n1 vào  1 ta lại f m  1      f mf f m   1, m

Đặt af 1 phương trình trở thành f m  1  f maf m   1, m  3

+ Với a2 Nếu a1 dẫn tới f 1 1(trường hợp giải trên) Do ta xét a1

Khi              

   

1

3 1 ,

2

f m a f m m

(46)

CH INH PH Ụ C OL YM PIC T O ÁN                      1

1 ,

2

f n a f n n

a a                        1

1 ,

2

n

f n a f n

a a              1 , n a

f n n

a

Đặt b a    

        1

\ 0,1 , ,

1

n

b

b f n n

b

Thay vào  2 ta   1 m b b        1 1, m b m b

 1 bm 1 b1 m  1 b

, m

     

1,

m m

b b b m  5

Thay m2 vào  5 ta b2   1 b 1 b1 2 b    1 0 b 1

bb1

Từ suy    

      1 , n

f n n , hay f n 0 n lẻ f n 1 n chẵn Thử lại thỏa mãn

+ Với a2 f 1 2

Thay n1 vào  1 ta fm 1  f m   1, m  6

Từ f 1 2  6 ta dùng phương pháp quy nạp tốn học chứng minh

  1, 

f n n n

Thử lại thấy thỏa mãn Vậy hàm số thỏa mãn yêu cầu đề

          

1

1

1

,

1 ,

n

n f n

f f n n với  n

Câu 33.Tìm tất hàm số f :  thỏa mãn f 0 2

 

  2     2  , , 

f x f x y f x f y x y  1 Lời giải

Thay x y vào  1 ta f f  0 2 0f   f  2 4 Thay x0 y1 ta f f  2  2 f 2  f  4 6

Ta chứng minh quy nạp với  x f 2x 2 2x   2 Theo  2 x0

Giả sử  2 tới xkk ,k0

Thay x0, yk vào  1 ta f f  2k  f   0  f 2kf2k12k 1

(47)

TẠ

P

CH

Í VÀ

LI

ỆU TO

ÁN

H

C

 

       

      

       

 

      

       

         

       

     

    

2 , ,

2 2 , ,

2 2 2 , ,

2 2, ,

2 2, ,

(2 ) 2,

f x f f x f x x x

f x f x f x x x

x x f x x x

f x x x x

f x x x Z x

f x x x Z

Dó (1) 1  f x  f x 2y2x2y 4, x y,   3

Ta chứng minh x số nguyên lẻ f x  số nguyên lẻ

Thật vậy, f x 2k, với k , thay x x 2k (với x lẻ) yk ta :

 

 2   2 2 2 4,  ,

f x k f x x k k x x lẻ

 

f x 2x2k  4, x ,x lẻ 2k2x2k  4, x ,x lẻ Do 4k2x4, với số nguyên lẻ x

Điều vơ lí 2x4 khơng phải lúc chia hết cho Như x lẻ f x  lẻ

Từ x lẻ x2y lẻ, dẫn đến f x 2y lẻ, xf x 2y chẵn Do f x  f x 2yxf x 2y2

Kết hợp với  3 ta xf x 2y 2 2x2y4 f x 2yx2y2,x y,  ,xlẻ

 

f xx2, x Thử lại thỏa mãn

Câu 34.Tìm tất hàm số f :  cho f f n   f n 2n  3, n  1

Lời giải

Giả sử tồn hàm số f n  thỏa mãn yêu cầu toán Cho n0, từ  1 có f f  0  f  0   3 f n 3  2

Nếu f 0 0 f f  0  f  0 0 mâu thuẫn  2 Vậy f  0 0 Nếu f 0 2 từ  2 ta có

 

  1  2    0  1  1    2 2.2 3   2 6

f f f f f f f f f

        ff f 2.1 3  f  1

Suy f 6  , loại

(48)

CH

INH

PH

C

OL

YM

PIC T

O

ÁN

Do f  0 1 Khi từ  2 ta có

 1    0  2  2    1 2.1 3   1 3

f f f f f f f

Ta chứng minh hàm cần tìm f n    n 1, n quy nạp toán học Thật Với n0thì f 0   1

Giả sử khẳng định tới n k k ,(  ) Tức là: f k( ) k

Với n k 1 ta có f k 1 f f k  2k 3 f k 2k 3 k1  k 1 Vậy khẳng định với n k 1

Theo nguyên lý quy nạp toán học, ta có f n    n 1, n Thử lại hàm tìm thỏa mãn yêu cầu toán

Câu 35.Chứng minh tồn hàm số f : * * thỏa mãn  

      , ,  *  

f m f n n f m b m n b  i

Lời giải Giả sử tồn hàm số f n  thỏa mãn yêu cầu toán Ta chứng minh f đơn ánh

Thật vậy, giả sử f n 1  f n 2 ,n n1, 2 *

Từ  i ta có f m f n   1  f m f n   2 n1 f m b  n2 f m b  n1 n2

Vậy f đơn ánh

Với  n *, ta có f f  1  f n  n f f  1 b  n f b b   f b f n   1

     

f n  1 b f nf , f đơn ánh      

f n 1 f nf  b a, với a f  1 b

Suy f n  f n 1  f 2  f 1  f 1    b a n, * Từ đó: f n  b naf n na b n  , *

Lúc f m f n    n f m b   f m na b    nm b a b  

 

m na b a b n ma ba b      

na2  n a2   1 a 1

, a 1 f n  * n b Suy f n    n b n, *b 

(49)

TẠ P CH Í VÀ TƯ LI ỆU TO ÁN H Ọ C

Câu 36.Hãy xác định tất hàm số f : *  * thỏa mãn đẳng thức:    1  2   3

f n f n f n f n a 1

Với a số tự nhiên thỏa mãn a1 số nguyên tố Lời giải Giả sử phương trình có nghiệm

Thay nn1 lúc  1 trở thànhf n  1 f n 2 f n 3 f n4a  2 Lấy    2  vế với vế ta được: f n 2 f n  f n 3 f n4 f n 2  3 Thay n lúc  3 trở thành: f 3  f  1  f    4 ff 3 

Thay n lúc  3 trở thành: f 4  f 2  f   5 ff 4  Thay n lúc  3 trở thành: f 5  f  3  f   6 ff  5  Thay n lúc  3 trở thành: f 6  f  4  f    7 ff  6 

Từ ta nhận quy luật đặc biệt tốn thay n số lẻ ta ln biểu thị được: f  3 f  1  f       4 f f nf 2n 1 f2n1  4

Nếu thay n số chẵn ta đặc biệt khác

 4   2      5 1  2  2 

f f f f f n f n f n  5

 Nếu f 1  f 3 lúc ta lậy tức có f2n1 f 2n1 lúc có vơ số số bé f 1 mà f  1 số hữu hạn Suy vô lí Tức f 3  f  1 Tương tự ta có f 4  f  2

 Nếu f  1  f  3 f 2  f 4 lúc f2n1 f 2n1và f 2nf2n2 Suy f  3  f 1 f 4  f 2 có vơ số ước số khác (vơ lí)

Từ có trường hợp

Trường hợp    

   

   

   

3 2

4 2 2

f f f n f n

f f f n f n

              

  , lúc

       

1

2

f f n

f f n

 

 



Thay vào phương trình  1 lúc f  1  f  2  f   3 4fa Mà lại có    

    f f f f    

  f  1  f  2  f    2 1f  af 1 1f 2 1 a Mà lại có a1 số nguyên tố nên ta có

           

1 2 2

2

2

2

2

f khi n k

f n

f a a n k

a n k

f a

f n

khi n k

f                                 

Trường hợp 2.    

    f f f f    

 lúc f  1  f2n1 với 

*

(50)

CH

INH

PH

C

OL

YM

PIC T

O

ÁN

Thay vào  5 ta    

       1    

2 2

4 n 2

f n f n

f f ff n f n

 

 

   



Lúc f 4  f 2 có vơ số ước số ngun dương đôi khác nhau, điều xảy nên phải có f  1 1

Thay n vào phương trình  1 ta có f 1  f 2  f   3 4fa Mà lại có f2n11

              

1

4 2 2 1 2

2

n k

f f a f n f n a f n a n

x n k

 

 

             

 

 Với x *

Trường hợp Nếu        

2

1

f f

f f

 

 



Lập luận tương tự     

1

1

2

khi n k

f n a n

y khi n k

 

   

  

 

*

y

Câu 37. Tìm tất hàm số f : *  * thỏa mãn f nt   a   f nan t a k

với t     

t

f nf f f n với a t, số tự nhiên tùy ý thỏa mãn k t2 1 a Lời giải

Đặt f n   n k an Ta chứng minh an 0 với số tự nhiên dương n

Từ giả thiết ta suy

 1        1  1

1 1

t a k n t a k

an

a f n an t a k f n n

a a a

    

        

    1

Với n2a 2 tk

  2    

n t a k

n t a k a ak k k f n n k

a

  

             

 Như với n2a  2 tk f n   n k

Khi với n2a 1 2kt k t  f n    n k 2a2kt2k t 2a 2 tk lúc ta có f f f   2  f f n    k f n  2 k1

Tiếp tục làm lần thứ t1 lúc ta có  t    1 1 2

fn   n t k  a tk

Và cuối f t  nf f t1 n  f t1  n    k n t k 1

(51)

TẠ

P

CH

Í VÀ

LI

ỆU TO

ÁN

H

C

a1n k a  nan  t a 1k n t k    1 97n 2 an  2

Từ  1 ta có  1  1   2 1 1 2 1 1

1

n n

k t tk n

a a a k n t a k a k t a

a a

 

             

 

Mà từ  2 ta có

 1  1  1  1  1

1

n n n n

t

a a a k t k t a k a a t a a

a

               

 Từ ta có an0

Xét với n t 1k Ta chứng minh phần toán phương pháp quy nạp Giả toán đến m n Khi

 t   1  t 2  2   t   3   2

fn t  kfn t  kfn t  k   f n n

   t 1  1  t  1    1     1   1 

f nf fn t kf n t ka n t k   t a k a f n t k

Theo giả thiết quy nạp f n(  (t 1) )k   n (t 2)k

Do f n an a t  1k tk ak k an a t      2k n  t 2k n k  Vậy f n  n k Thử lại thấy thỏa mãn

Câu 38.Cho hàm số f :  thỏa mãn:

   

         

   

2 2

,

f n f n f n f n f n

n

f n f n

        

 



Tìm n cho f n 2009

Lời giải Giả sử tồn hàm f thỏa mãn đề

Vì 2  f n  số nguyên dương lẻ f 2n1 f 2n 1 số nguyên dương lẻ 2 1  2  

f n f n f n

    

2 1  2  

f n f n f n

         

      

2 1 2

f n f n

f n f n f n

   

  

    



   

 

           

2 2 2

2 2 3

f n f n f n f n

f n f n f n f n

  

    

 

 

    

 

Ta chứng minh f n  f n 1 (1) Với n 1 f  1  f  0 2 f  0

Giả sử f n  f n 1 tới k k N(  *) f 0  f  1  f 2  <f k 

 Nếu k chẵn Đặt k2m m  *  f k 1 f2m1 f 2m 2 f 2mf k   Nếu k lẻ Đặt k2m1m 

 1 2 2  1    2 2 1  

f kf mf mf mf mf mf k

      

(52)

CH

INH

PH

C

OL

YM

PIC T

O

ÁN

Do f  0 3 0f   f 0 0, f 1 2;f 2 3 1f  6

 3  2 8,  13  12 3  74,  27 13  224

ff   ff   f   ff  

 53 13  668, 107 53  2006, 108 27  2016

ff   ff   ff

107 2009 108

ff

 

Do f n 2009 n0;1; 2; ;107

Câu 39.Tìm tất hàm số f :  thoả mãn:

       

1 1, , ,

3 f xy 3 f xzf x f yz 9 x y z

Lời giải

Cho x y z  0          

2

1 1 1

0 0 0

3 f f f f f

 

        

 

Cho x  y z          

2

1 1 1

1 1 1

3 f f f f f

 

        

 

Cho y z 0      0

3 ff x f

Do  0

f  nên   1,

f x   x  1 Cho y z 1, ta có        1

3 f x 3 f xf x f 9

Do  1

f  nên   1,

f x   x  2 Từ  1  2 ta   1,

3

f x   x

Câu 40.Cho n n2 hàm số f :  cho:  nn     ; ,

f xyxf x f f yx y  *

a) Giả sử f 20020 Tính f 2002  b) Tìm hàm số f

Lời giải a) Từ  * ta

 Với x0;f y  f f y  , y

 Với x1;y0 : f f  0  0 f  0 0

 Với x1,y : f 1y f  1  f y   1 Do đó, chứng minh quy nạp ta f n nf  1 , n  2

(53)

TẠ

P

CH

Í VÀ

LI

ỆU TO

ÁN

H

C

Từ    2 , ta f n nf  1 , n  4 Đặt  1 p;p ,q *

q

f    ta  n * n chia hết p nên nf 1 

Do ta               

 

2

1 1

1

f

f f n f n n f nf f f

f

 

        



Do đó, từ  4 ta f20021 hay f 20020 (loại) Vậy f20022002

b) Từ  * ta y0 : f x nxn1f x , x  1  Nếu n chẵn: :    1 ,    1  

n n

n n

f x f x

x f x f x f x

xx

      

 Nếu n lẻ từ  *  1 ta f xny   f xnf y   2

Suy         

   1  

0

n n

n n

n n

f x f x

f x f x f x f x

x

x  

 

        

Do f   x f x , x

Từ  2 , chứng minh quy nạp ta f px npf x n , p , x Có  p * :f pxn f px n  pf x n

Vậy f px npf x n ,   p , x  3 Từ  3 ta có uu ,v *

v

    ta

1

1

n

n

n n

u u v

f f uv f

v v v

 

    

 

   

     

Mà  1 1  1

n n

n n n n

f u

f f v f f

v v v v

     

      

   

 

Vậy f u u v n f n1 u f  1  4

v v v

   

 

 

Ta có f  1 0 hay f  1 1 từ  4 suy f x   0, x hay f x   x x, Thử lại thỏa mãn  *

Vậy f x   0, x

Câu 41 Tìm tất hàm số f :  thỏa mãn

 3   2  3  , ,

f x y zf xf yf zx y z

Lời giải

Đặt P u v t , ,  phép x u, y v, z t, ta có:

P0,0,0 f  0 0

(54)

CH

INH

PH

C

OL

YM

PIC T

O

ÁN

P0, ,y z  

 

1 1

f f

 

 

Với f 1 1, ta chứng minh f x xf2 1 , x

Giả sử điều phải chứng minh đến x k k  , : f k kf2 1 Ta chứng minh đến x k   1 k , thực vậy:

 1   2 1 2 1 2  1 1  2 1

f k  f kfkffkf

Với f 1 0, ta chứng minh quy nạp f x 0  x

Thế vào giả thiết ban đầu, ta nhận hai hàm thỏa đề: f x 0 f x x  x

Câu 42.Cho hàm số f : *  * thỏa mãn đồng thời hai điều kiện:

a) f ab  f a b f a b ,   ,  với a b,  *,a b ;    a b, , a b, bội chung nhỏ nhất, ước chung lớn hai số nguyên dương a b, ;

b) f p q r    f p f q f r  với số nguyên tố p q r, ,

Tính giá trị f 2013? Kí hiệu * tập hợp tất số nguyên dương Lời giải

Đặt f 2 a f,  3 b Khi ta có đẳng thức sau:

 7 2 3  2  3

ff    ffa b

 8 2 3  2  3

ff    ff  a b

 16 7 2 7   2 2 

ff    ffa b  a ab

 16     2 

ff fa ab

Do ta có 5a2b a 22ab 1 Mặt khác ta có đẳng thức sau:

 12 2 7  2  3  7

ff    fffab

 12    2 6  2 2 3

ff fa f    a

Suy 3a2b3a2  2

Từ    1 , ta có    

2

5 2

7, 8

3

3

a b a ab a

f f

b

a b a

     

     

  

 

 

Ta có 2003 số nguyên tố nên

2013 2003 7 2003  3  7 2003 10

ff    ffff   3

2025 2003 17 2003  5    17

ff    fff

 9 3  5 2  5  2  5

ff   f    fff

 17 7 3 7   3 17

ff    ff

(55)

TẠ

P

CH

Í VÀ

LI

ỆU TO

ÁN

H

C

Mặt khác f2025 f 9.9.25 f  9 f 9.259 5.5.9f  9 5f    f 45            

     

 

9 3.15 45 15 45 45.3 2025

f f f f f f

f f f

    

   

Do f 20252025, kết hợp với (5) ta f 20032003 Do từ đẳng thức  3 ta f 20132013

Câu 43 Đặt Ff : 0,1    0,1 n2 Tìm giá trị nhỏ c thỏa mãn điều kiện

   

1

0

n

f x dx c f x dx

 

Với f Ff hàm liên tục

Lời giải

Ta có   1   1       

0 0 0,1

n n

n

f x dx n y  f y dy n x  f x dx n f x dxx

    , c n

Với p0, ta chọn hàm f x xp, đó: c n p 1

n p

 

 Do đó: lim  1

p

n p

c n

p n



 

Vậy c n , lại có n2 nên giá trị c cần tìm

Câu 44 Tìm tất hàm f : 1,1  liên tục, thỏa mãn:

 

2

x

f x f

x

 

   ,  x  1,1

Lời giải

Đặt   2

x g x

x

 Bài toán trở thành: f x  f g x   với   1 x

Ta chứng minh toán nhỏ: Gọi dãy số ang an1 Khi đó, với giá trị dương

1

0a 1, ta có: limnan 1

Chứng minh. Với số thực 0 a 1, ta có: 2

1

a a

a

  a g a  

Lại có, với 0 a 1, ta thấy 0g a 1

Vì  an n1 dãy tăng nghiêm ngặt có giới hạn Đặt limnanL Ta có:  

2 1

1

L

L g L L

L

   

(56)

CH

INH

PH

C

OL

YM

PIC T

O

ÁN

f hàm liên tục nên:    

lim 1

hfhf , nói cách khác với 0 tồn 

sao cho với 0  h : f 1    f

Đặt x  1 , từ chứng minh ta có hàm x g x g x,  , 2 , tiến đến Vậy tồn số nguyên dương k cho g xk   1 Điều đồng nghĩa:

 

k   1

f g xf

f x  f g xk  nên f x    f  , chọn giá trị vơ bé, ta được:

   1    1

f xf   f xf với 0 x Chứng minh tương tự, ta f x  f 1 với   1 x

f hàm liên tục nên          

lim 1

xf xfff   f

Vậy f x c với c số

Câu 45 Có thể tồn hay khơng hàm số f :  , liên tục thỏa mãn điều kiện: Với số thực x, ta có f x  số hữu tỉ f x 1 số vô tỉ

Lời giải

Giả sử tồn hàm số liên tục f :  thỏa mãn điều kiện:

     

: \ *

x f x f x

     

Xét hàm số g x  f x  1 f x , h x  f x  1 f x 

Khi g h hàm số liên tục Ta có g h khơng thể đồng thời hàm Thực vậy, giả sử g x C h x1,  C2 Khi đó:

   

2f xCCf xC (C số) Vì với   f    f  1C, điều mâu thuẫn với  *

Giả sử h khơng hàm hằng, khơng tính tổng qt, tồn x x x1, ,2 x2

cho h x 1 h x 2

Lúc tồn số hữu tỉ r h x   1 ,h x2  Ta có h x 1 r h x    2 r0

Lại có h x r hàm liên tục với x thuộc nên phương trình h x  r có nghiệm, tức tồn x0 cho h x 0 r, từ f x 0 1 f x 0 r

r nên f x 01 ,  f x0 đồng thời hữu tỉ đồng thời vô tỉ, điều mâu thuẫn với  *

(57)

TẠ

P

CH

Í VÀ

LI

ỆU TO

ÁN

H

C

Câu 46 Tìm tất hàm số f :  thỏa mãn điều kiện f x  f t  f y  f z  với số hữu tỉ x y z t   x y z t, , , theo thứ tự lập thành cấp số cộng

USAJMO 2015

Lời giải

Do x y z t, , , theo thứ tự lập thành cấp số cộng nên x t y z   , kết hợp với phương trình

cho ta f x  f y z x    f y  f z   1 với x y z  x, y, z theo thứ tự lập thành cấp số cộng

Thay x0 vào (1) ta f 0  f y z   f y f z  với 0 y z Đặt g x  f x  f 0 ta g y z  g y   g z   0 y z  2 Với số nguyên dương n3, ta tìm cách biểu diễn g n  theo g   1 ,g

Ta có g 3 g     1 g ,gg     3 gg 2 1g , dùng quy nạp ta chứng minh g n g  2  n2    g

Lấy 2 y z số nguyên dương ta được:

   2  2    1 ,   2 2  1 ,    2  2  1

g ygyg g zg  z g g y z gy z  g ,

Thay vào  2 ta

 2  2  1  2  2     1 2  1  2  

gy z  ggygg  z ggg  *

Kết hợp với  3 ta g n ng 1 Ta biểu diễn g nx  theo g x g x   ,

Bằng quy nạp ta dễ dàng chứng minh g nx g x  2  n2    g x Lấy 2 n m số nguyên dương, thực thao tác  * ta

  2 2     2 2     2 2    2  

g xm n  g xg xng xg xmg xg xg x

Kết hợp với  4 ta g nx ng x   5

Vậy với m n, nguyên dương: g m  ng m g m mg 1 g x  xg 1

n n n

   

      

    , với x

 Do f x  f t  f y  f z g x   g tg y   g z    x y z t lập thành CSC Với x0 bất kỳ, ta xét CSC x  0 y z ta

       0    1  0  1    0  1  1  1

g xg tgg zg xtggzgg xgzgtgxg

(58)

CH

INH

PH

C

OL

YM

PIC T

O

ÁN

Câu 47 Giả sử r s,  hai số cho trước Tìm tất hàm số f :  thỏa mãn điều kiện f x f y    f x r    y s, x y, 

Romania 2006

Lời giải Thay x0 vào phương trình ban đầu ta

 

    ,  1

f f yf r  y s  y

Thay y f y  thu được:

 

 

      , ,

f x f f y  f x r  f y  s x y

 

      , ,

f x f r y s f x r f y s x y

         

 

      , ,

f x r y f r r s f x r f y s x y

           

 

      , ,

f x y f r r s f x f y s x y

         

Đặt af r  r s thay vào phương trình ta được:

      , ,  2

f x y a   f xf y  s x y

Thay y0 vào phương trình (2) ta được: f x a   f x  f  0 s,  x  3 Từ    2 , ta được:

   0     , ,

f x y  f  s f xf y  s x y

   0    , ,

f x y f f x f y x y

      

   0    0    0 , ,

f x y f f x f f y f x y

        

Đặt f x  f 0 g x , x thay vào phương trình ta      , ,  4

g x y g xg yx y

Từ  4 ,theo kết phương trình hàm Cauchy ta g x bx x b, ,  Từ cách xác định hàm số g ta f x bx f  0 bx c  5

Từ  5 thay lại vào phương trình ban đầu ta được:

    , ,

b x by c   c b x r    y s c x y

2 , ,

bx b y bc c bx y br s c x y

          

2 1

1

b r s b

b bc c br s c

c r s

   

 

   

   

   

  

(59)

TẠ

P

CH

Í VÀ

LI

ỆU TO

ÁN

H

C

Câu 48 Tìm tất hàm số f :  cho với tất số nguyên a b c, , thỏa mãn a b c  0, đẳng thức sau đúng:

 

 2   2   2            

2 2

f af bf cf a f bf b f cf c f a

IMO 2012

Lời giải

Lời giải Tôn Ngọc Minh Quân

Giả sử hàm f :  thỏa mãn điều kiện đề

Cho a b c  0, ta f 0 0

Cho a n b ,  n c, 0n ta f  n f n  Đặt f   1 t t 

Cho a2,b 1,c 1 ta có f  2 0 f 2 4tTrường hợp 1.f  2 0  f 3 t

Ta có f  4 2f 2 2 f(2)2 2f    2 f 2f   2 f 2f   2 ff  4 0 Giả sử f 2i 0, f2i1t 1 i k

 

 2   2   2  

2 2 2

f k f k ff k

      

Ta có f 2k32f  2k 2f  3 2 2f  3 f 2k 3 f2k3 f  3 t

Vậy f  2i 0,f 2i1  t, i Nf 2i 0, f2i1  t, iTrường hợp 2. f 2 4t t  ,t0

Ta có f  3 2 f  2 2f  1 2 2f   1 f 2f   1 f 2f   2 f Suy f 3 t f 3 9t

a) f 3 9t,f  2 4t,f  1 t Ta chứng minh f n n t2 , n * Thật mệnh đề với n1,2,3 Giả sử mệnh đề đến n3

Ta có f n 12f n 2f  1 2 2f   1 f n 2f  1 f n12f n f n   1  

 2  2    2 2 2

1 1

f n t n f n t n

       

   2

1

f n t n

    f n  1 t n12 Giả sử f n  1 t n12  f n 1

Ta có f n 12 f 2 2 f n 12 2f   2 f n12f   2 f n12f n 12  

 2      

2 2 1

f f f n f n

    

 2  2

2 2

16t 2t n t 16t 16t n

     

Đó điều vơ lý (vì n3)

(60)

CH

INH

PH

C

OL

YM

PIC T

O

ÁN

 

 2   2   2            

4 2 2 2 ( ) 2

ffff ff ff f

 4

f

  f 4 16t

Giả sử f 4 16t Ta có f 4 2 f 3 2f 1 2 2f   1 f 2f   3 f 2f    1 f

2 2 2

256t 2t 32t 32t 2t

     192t2 0

(vô lý) Vậy f  4 0

Ta có f  5 2 f  4 2f 1 2 2f   1 ff(5) f(1)t,  

 2   2   2    

6 2

ffff ff  6  f 2 4t,

 

 2   2   2    

7 3

ffff ff(7) f(3)t,

 

 2   2   2

8 4

fff   f 8 0

Bằng phương pháp quy nạp toán học ta chứng minh 4 1

f i t  i N; f4i3t  i

 4

f i   i N; f4i24t  i

Thật giả sử f  4k 0,f4k1t f, 4k24 ,t f 4k3 t k N 

Ta có f 4k12f 4k 2f 1 2 2f    1 f 4k 2f  4k f 4k12f   1 f 4k1 4 1  1

f k f

  

 

 2   2   2            

4 2 4 2

f k  f kff f kf k f k  f f k

4 2  2

f k f t

   

 

 2   2   2            

4 3 4 3

f k  f kff f kf k f k  f f k

4 3  3

f k f t

   

 

 2   2   2            

4 4 4 4 4 4

f k  f kff f kf k f k  f f k

4 4  4

f k f

   

Suy f 4i 0,f4i1t f, 4i24 ,t f4i3t t  ,t0 i

Ngược lại, giả sử hàm f :  thỏa mãn f 2i 0,f 2i1 t t  với i Giả sử a b c, ,  ,a b c  0 Suy số a,b,c có số chẵn

+) Nếu a,b,c chẵn f a  f b  f c 0  

 2   2   2            

2 2

f a f b f c f a f b f b f c f c f a

     

+) Nếu a chẵn b,c lẻ f a 0, f b  f c t  

 2   2   2 2

2

f a f b f c t

   

           

 

2 f a f bf a f cf b f c 2t

 2   2   2            

( ) 2

f a f b f c f a f b f b f c f c f a

(61)

TẠ

P

CH

Í VÀ

LI

ỆU TO

ÁN

H

C

Tương tự b chẵn a,clẻ c chẵn a,blẻ ta có:  

 2   2   2            

2 2

f af bf cf a f bf b f cf c f a

Vậy hàm f :  cho f 2i 0,f 2i1 t t  với i thỏa mãn điều kiện đề

 Xét hàm số f :  thỏa mãn f n n t t2   ,t0 n Giả sử a b c, ,  thỏa mãn a b c  0

Ta có f a a t f b2 ,  b t f c2 ,  c t2

Suy f a 2 f b 2f c 2 a4b4 c t4

a b c   0 a2 b2c2  2ab2bc2ca

 

4 4 2 2 2 2 2 2 4 2 4 2 4 2 8

a b c a b b c a c a b b c a c abc a b c

           

4 4 2 2 2 2 2 2

a b c a b b c a c

     

 

 2   2   2 2 2 2 2 2 2

2 2

f a f b f c a b t b c t a c t

     

           

2f a f b 2f b f c 2f c f a

  

Vậy hàm f :  cho f n n t t2   ,t0 n thỏa mãn đề  Xét hàm f :  thỏa mãn

4 1 , 4 2 , 4 3 ,  4  , 0

f i t f i  t f i t f itt  i

Giả sử a b c, ,  cho a b c  0 + Nếu a4i i    b c mod 4 

+ Nếu b,cđều chia hết cho f a  f b  f c 0  

 2   2   2             

2 2

f a f b f c f a f b f b f c f c f a

      

+ Nếu b2 mod 4  c2 mod 4  f a 0,f b 4 ,t f c 4t  

 2   2   2 2

32

f a f b f c t

   

           

2f a f b 2f b f c 2f c f a 32t

 

 2   2   2            

2 2

f a f b f c f a f b f b f c f c f a

     

+ Nếu b1 mod 4  c3 mod 4  f b t f c,  t  

 2   2   2 2

2

f a f b f c t

   

           

2f a f b 2f b f c 2f c f a 2t

 

 2   2   2            

2 2

f a f b f c f a f b f b f c f c f a

     

+ Nếu a1 mod 4 , b0 mod 4  c 3 mod 4 , tương tự ta có:  

 2   2   2            

2 2

f af bf cf a f bf b f cf c f a

(62)

CH

INH

PH

C

OL

YM

PIC T

O

ÁN

 

 2   2   2            

2 2

f af bf cf a f bf b f cf c f a

+ Nếu a1 mod 4 , b2 mod 4  c1 mod 4    ,   ,  

f a t f b t f c t

    f a( ) 2 f b( ) 2  f c( )2 18t2

            2 2

2f a f b 2f b f c 2f c f a 8t 8t 2t 18t

 

 2   2   2            

2 2

f a f b f c f a f b f b f c f c f a

     

+ Nếu a1 mod 4 , b1 mod 4  c 2 mod 4 , tương tự ta có:  

 2   2   2            

2 2

f af bf cf a f bf b f cf c f a

+ Nếu a2 mod 4 , b0 mod 4  c 2 mod 4  a2 mod 4 , b1 mod 4 

 

1 mod

c ; a3 mod 4 , b0 mod 4  c1 mod 4 hoặc a3 mod 4 ,

 

1 mod

bc0 mod 4 , tương tự ta có:  

 2   2   2            

2 2

f af bf cf a f bf b f cf c f a

+ Nếu a3 mod 4 , b3 mod 4 , c2 mod 4 thì f a  f b t f c,  4t  

 2   2   2 2

18

f a f b f c t

    ; 2f a f b   2f b f c   2f c f a   18t2

 

 2   2   2            

2 ( )

f a f b f c f a f b f b f c f c f a

     

Vậy tất hàm f :  thỏa mãn đề là:

:

f  : f 2i 0,f 2i1t t   i

:

f  : f n n t t2   ,t0 n

:

f  : f 4i 0, f4i1t f, 4i24 ,t f 4i3t t  ,t0 i

Cách

Thay a b c  0 vào phương trình ban đầu ta được:

 2  2  2              2  2  

0 0 0 0 0 0

ffff ff ff ffff

Thay b a c, 0 vào phương trình ban đầu ta được:

 2  2  2            

0 2

f afaff a f  a fa ff f a

f a 2 f a 2f a f    a f a 22f a f      a fa  0 f a  f  a

Ta viết lại phương trình ban đầu dạng:  2           2

2

f cf c f af bf af b

                    

2

2 4

2

f a f b f a f b f a f b

f c f c f a b     

     

         

f a b f a f b f a f b

    

Nếu f b 0: f a b   f a  f a mod b

(63)

TẠ

P

CH

Í VÀ

LI

ỆU TO

ÁN

H

C

Trường hợp f 1 0, ta có f 2  f 1  f 1 2 f    1 ff  2 0 or f  2 4 1f   Ta xét hai trường hợp nhỏ:

Trường hợp 2.1: f 1 0,f 2  0 f x  f x mod 2 f x  f(1) x lẻ f x 0 x chẵn

Trường hợp 2.2: f 1 0,f  2 4 1f   f 3  f  2  f 1 2 f   2 f

 3 1  1   3  1 1 

f f f f f f

     

 Nếu f  1 0, f 2 4 ,f    ff  1 :  4  1  3    1

f f f f f

    f 4  f  2  f 2 2 f   2 f  4  1

f f

  f 4 16 1f   4    mod 

f f x f x

   

 Nếu f  1 0, f 2 4 ,f    f 9 1f :

 4 1 3  1  3    1 16 (1)

f f f f f f f

       f  1

f  4  f  2  f  2 2 f  2 16 1f  0. f  4 16 1f   Nếu x4, f x  f 1 x2

Dùng quy nạp, ta chứng minh: f x  f 1 x x2

 Nếu x m ., f x  f 1 x x2 , với số giá trị m Giả sử điều phải chứng minh với m k :

            2

1 1

f k f k f f k f f k

       f  1 k12

f k 1 f k  1 f 2 2 f k 1  ff  1 k12 f  1 k32

    2

1 1

f k  f k

Vậy điều phải chứng minh với m k 1

Vì với m4, theo quy nạp toán học ta kết luận f x  f  1 x x2

Câu 49 Tìm tất hàm f g, :   có đạo hàm  thỏa mãn    

' g x

f x

x

  ; g x'  f x 

x

   x

Lời giải Ta có x f x.  g x 'x f x ' g x'  f x g x 

       

0,

g x f x

x f x g x x

x x

 

       

 

x f x    g x   a xf x g x  a

(64)

CH

INH

PH

C

OL

YM

PIC T

O

ÁN

Tương tự ta có f x   g x ' 0, x

x

 

  

 

   f x   g xbx x, 0  x0  2 Từ    1 ,   

2

a

f x bx

x

 

   

 ;  

1

, 0, ,

a

g x bx x a b

x

 

      

 

Câu 50 Tìm tất hàm f : *  có đạo hàm * thỏa mãn

      , *

f xyf xf yx y  1

Lời giải Lấy đạo hàm hai vế  1 theo biến x y, ta có

    *

' ' ,

yf xyf xx y 

    *

' ' ,

xf xyf yx y 

   

' '

x f x y f y

  x y, *

 

'( )

x f x a

  x *

   f x a.lnx bx R*

  Thử lại  b

Vậy f x a.lnx  x *

Câu 51 Tìm tất hàm f :  thỏa mãn  

  1 

f f n    n b n

trong b số nguyên dương chẵn

Lời giải Giả sử f thỏa mãn đề Dễ thấy f đơn ánh

Trong  1 thay n f n  ta f f f n     f n b Suy f n b   f n   b n

Vậy m qb r  ,0  r b f m  f qb r   f q 1b r   b f r qb Bây ta cần xác định hàm f tập A0,1, 2, ,b1

Xét x A , đặt f x y f y  x b

Giả sử y qb r  f y  f r qb suy x b f r   qbqb x b  2b nên q0 q1

Do x A

i) f x  r b r A f r,  ,  x

ii) f x r r A f r,  ,   x b

Vậy hàm f xác định sau

     

 i i

f qb r qb f r

f a b

f b a b

   

 

  

(65)

TẠ

P

CH

Í VÀ

LI

ỆU TO

ÁN

H

C

Câu 52 Tìm tất hàm f :    thỏa mãn:

i) f xf y  yf x  ,x y   1

ii) lim  

xf x

IMO 1983

Lời giải Thế x1 ta f f y  yf 1

Nếu f a  f b , af  1  f f a   f f b  bf  1  a b f  1 tiến tới số nguyên dương Vì hàm f đơn ánh

Thế x y ta đượcf xf x  xf x( ) Vậyxf x  điểm bất động hàm f

Khi đó, thay y1 thu đượcf xf  1  f x  f 1 1 x khác nên điểm bất động hàm f Ta chứng minh điểm bất động hàm f

Giả sử tồn a,b điểm bất động f , ta có f a a f b b Khi đó, x a y b ,  vào  i ta f ab  f af b  bf a ab Suy ab điểm bất động f

Thế x 1,y a

a

  thu f  1 f a f f a  af f 1

a a a a a

       

           

       

Vậy

a điểm bất động f

Nếu f a a với a1, f a n điểm bất động f , mâu thuẫn  ii Nếu f a a với 0 a 1, f 1n 1n

a a

  

 

  điểm bất động f , mâu thuẫn  ii Từ kết ta thu điểm bất động 1, suy xf x  f a 

x

  

Câu 53 Chứng minh tồn song ánh f :   cho 3          ,

f mn m n   f m f nf mf nm n 

IMO Shortlist 1996

Lời giải

Xét hàmg:  1 1 thỏa mãn:   1

x

g xf   

  ta có g song ánh

  

 3  3 1 3  ,

g mn g mg n m n 

Thật g3m1 3 n1g3 3 mn m n  14 3fmn m n  1

       

            

4 4f m f n f m f n 4f m 4f n g m3 g n3

         

(66)

CH

INH

PH

C

OL

YM

PIC T

O

ÁN

Như thế, ta cần song ánh g đủ,   3 1

g x

f x    thỏa mãn đề

Xét P P1, tập số nguyên tố dạng 3k1, 3k2 tương ứng Q Q1, tập số

nguyên tố dạng 4k1, 4k3 tương ứng, ta xét song ánh h P: 1P2 Q1Q2 Sao cho h P 1 Q h P1,  2 Q2 xác định g sau:

g 1 1

 nguyên tố g n h p i

Rõ ràng song ánh g thỏa mãn đề

Câu 54 Tìm tất hàm f :  thỏa:  

 

      

3f f f n 2f f nf n 6 ,n  n

Lời giải Đặt akf nk  f f  f n  (lặp k lần) Ta lặp dãy 3ak2ak1ak2 6ak3 với n

Xét tập Sa a a0, , ,  tập số nguyên dương nên tồn phần tử có

giá trị nhỏ Đặt j số cho aj giá trị nhỏ tập S Ta có bất đẳng thức 3ak2ak1ak2 6ak3 6aj

Đẳng thức xảy akak1 ak2 aj

Lại có đặt k j đẳng thức xảy ra, ta có ajaj1 aj2 aj3

Tịnh tiến giá trị ta thu akaj với k j

Thực tương tự cho tịnh tiến lùi, lưu ý 3 2

k k k

k

a a a

a       , ta thu

k j

aa

với k j Vậy akaj với k0

Từ chứng minh dẫn đến a1 a0 hay f n n , n

Câu 55 Tìm tất hàm số f : 0;   0; thỏa mãn điều kiện:  

     , 0;   

f f xyf yf xx y 

Lời giải

Với t0;, ta chọn tùy ý x0 cố định u f x  0 , v tf x  0

   

f u t

f v

 Ta thay  1 y

 1

f y ta có     

   

1 f x

f f x f

f y f y

 

  

(67)

TẠ

P

CH

Í VÀ

LI

ỆU TO

ÁN

H

C

Hay  

       2 

f x

f f y f f x

f y

 

 

 

 

Trong  1 , thay y  1

f x ta           

1

1 0;

f f x f x

f x

   

Từ    2 , suy  

     1  

f y f x

f f

f y f x

 

 

 

 

Do f f u   f   1 f v 

f v f u

 

 

 

 

Do vậy, f t  f  1 t 0; 

t

   

Thử lại, hàm cần tìm f x  a x 0; 

x

    a0 số

Câu 56 Chứng minh tồn hàm số f xác định tập số thực dương, nhận giá trị thực dương thỏa mãn f f x  6x f x  

Putnam 1988

Lời giải

Với số thực dương x0 cố định, ta xây dựng dãy  fn n1 sau:

    

1 0, , n n

fx ff x f   f f x

Khi đó, từ đẳng thức giả thiết ta suy dãy  fn n1 thỏa mãn phương trình truy hồi

2 1,

n n n

f   ff

Hay fn2 fn16fn0

Đến đây, giải phương trình đặc trưng dãy  fn n1, ta hai nghiệm 3 Do đó, fn a 2n  b  3 ,n đó, số a, b tìm phụ thuộc vào f f1,

Tuy nhiên, b0, tồn nđủ lớn cho fn0 (ta thấy dễ dàng

cách chọn n chẵn đủ lớn b0, chọn n lẻ đủ lớn b0) Do vậy, b0 Thành fn  a 2n

Suy f f x  0 a.2 ,3 f x 0 a.22, thay hai giá trị vào đẳng thức

 

   0

f f xxf x

Ta 2a x Dẫn đến f x 0 2x0

(68)

CH

INH

PH

C

OL

YM

PIC T

O

ÁN

Câu 57 Hàm số f :  thỏa mãn đồng thời điều kiện sau:

      

       

     

: , : 2 ,

:

i f f n n n

ii f f n n n

iii f

  

    

Tìm giá trị f1995 , f 2007

Olympic Ukraine 1995

Lời giải

Cũng nhận xét lý luận ví dụ trước, ta đưa đến f n  phải có dạng: f n an b Khi điều kiện  i trở thành:a n ab b n n2     ,

Đồng hệ số, ta

2 1 1 1

0

0

a a

a

b b

ab b

  

   

 

      

 

Với

0

a b

   

 ta f n n Trường hợp loại khơng thỏa mãn  ii

Với

0

a b

    

 ta f n   n b

Từ điều kiện  iii cho n0 ta b1 Vậy f n   n 4 

Hiển nhiên hàm số thỏa mãn điều kiện toán

Ta phải chứng minh f n   n hàm thỏa mãn điều kiện toán Thật giả sử tồn hàm g n  khác f n  thỏa mãn điều kiện toán Từ  iii suy f 0 g 0 1

Từ  iii suy f 1 g 1 0

Sử dụng điều kiện    i , ii ta nhận g g n  g g n  22  n Dno g g g n   g g g n   22  n

Hay g n g n 22  n

Giả sử n0 số tự nhiên bé làm cho f n 0 g n  0

Do f n  thỏa mãn  4 nên ta có

       

   

0 0

0

2 2

2

g n g n f n f n

g n f n

      

   

Mâu thuẫn với điều kiện n0 số tự nhiên bé thỏa mãn  5

Vậy f n g n ,  n

(69)

TẠ

P

CH

Í VÀ

LI

ỆU TO

ÁN

H

C

Vậy f n  1 n nghiệm Từ tính f1995 , f 2007

Câu 58 Tìm f : 0,1  thỏa mãn:

        , ,  0,1

f xyzxf xyf yzf zx y z

Lời giải Chọn x y z  ta f x 3 3xf x 

Thay x y z, , x2 ta f x 6 3x f x2  2

Mặt khác f x  6  f x x x .2 3xf x x f x2  2 x f x3  3 Hay 3x f x2  2 xf x x f x2  2 3x f x4  

     

2

2x f x xf x 3x f x

    2 3  ,

2

f x xf x x

   

Thay x x3 ta    

9

6 3 ,

2

x

f x   f x  x 3  2 13  ,

2

x

x f xxf x x

   

     

3

2 3

3 , 0,

2

x x

xf xxf x x f x x

       

Vậy f x 0 với x 0;1

Câu 59 Tìm tất hàm f xác định thỏa mãn đồng thời điều kiện sau:

           

2 ,

1

f n f k n f k n f n f k k n

f

    

 

 

Lời giải

Cho k n 02f2 0 2 0f  3f2 0  f  0  0 f 0  2

Nếu f 0 0 chọn n0 ta được:2f k 0 f k 0 với k Chọn k1 ta f 1 0 mâu thuẫn với giả thiết

Vậy f 0  2

Chọn n1 ta phương trình 1f  f k 1 2f k 13 1f   f k ,k

     

2f k 2f k 3f k , k

     

Đặt xkf k ta có phương trình sai phân 2xk13xk2xk1 0

Phương trình đặc trưng 2 2             Vậy   12 2

2

n n

f ncc  

(70)

CH

INH

PH

C

OL

YM

PIC T

O

ÁN

Ta tìm c c1, từ điều kiện f 0  2,f 1 1

Dễ tìm c1 0,c2  2

Vậy  

2

n

f n    

 

Câu 60 Tìm tất hàm số f : *  * thỏa mãn đồng thời hai điều kiện sau:  

 

   

* *

*

2 , ,

1 ,

f f n n k n k

f n f n n

     

 

   



Lời giải Giả sử có hàm f thoả mãn điều kiện    1 , Ta chúng minh hàm f đơn ánh

Thật vậy, với m n,  *và m n , có f m  f n   1 suy  

     2

f f mf f n  m k n  km n (vô lý)

Suy f đơn ánh Do  2  f n 1 f n  với n * suy  1  

f n  f n  với n *  

     1   

f f n f f n f f n

      với n *

 

    

f f n f f n

    với n *

1 2

n k n k

      với n *

 

     1

f f n f f n

    với n *

 1  

f n f n

    với n * ( f đơn ánh)    1

f n f n

    với n2 ; n * Truy hồi ta

   1

f n   n f với n *

 

    1  2 1 

f f n f n f n f

       với n *

 

2 2

n k n f

     với n *

 1

f k

   với n *

(71)

TẠ P CH Í VÀ TƯ LI ỆU TO ÁN H Ọ C

Câu 61 Tìm tất hàm số f :  thỏa mãn đồng thời hai điều kiện sau:  

 

2013 2016  4,

f

f f n n n

        Lời giải Giả sử có hàm f thoả mãn điều kiện    1 ,

Từ điều kiện  1 ta dễ dàng chứng minh f đơn ánh Từ  1 ta suy f f f n    f n 4 với n

   4

f n f n

    với n (3) Với n4k r với k ;r0,1, 2, 3

Từ  3 ta suy f4k r   4k f r   với k

 Tính f 1 Do 2013 4.503 1  nên f20132012 f 1 2016 f  1 4  Tính f 0 Ta có f f  0  4 f  1  f  0 1 (do f đơn ánh)

 Tính f 2 f 3 Giả sử f 2 4m r với mr0,1, 2, 3

Do  1 mà ta có f f  2   6  f f  2  f 4m r 4m f r  6; Mà f r 0m0 m1

+ Với m0, f r 6 f f  2  f r  f  2 r + Với m1, f r 2 f  2  4 r

Trường hợp 1. Xét m0  

    

6

2 0,1, 2,

f r

f r r

 

  

 + Với r0  

  f f    

 vô lý f  0 1 + Với r1  

  f f    

 vơ lý f 1 4 + Với r2   2

2 f f    

 vô lý

+ Với r2     f f    

 vơ lý

Vậy m0 ta có f  0 1;f  1 2;f 2 3;f 3 6

Trường hợp 2. Xét m1  

    

2

2 0,1, 2,

f r

f r r

  

  



(72)

CH

INH

PH

C

OL

YM

PIC T

O

ÁN

+ Với r2    

2 2

f f

 

 vơ lý

+ Với r2    

2

f f

 

 

 vô lý

Vậy m =1 ta có f 0 1;f 1 4;f  2 7;f 3 2

Suy  

 

 

 

 

1 , n mod , n mod , n mod , n mod

n n f n

n n

 

 

 

  

 

  

Thử lại: f n  thỏa mãn đồng thời điều kiện (1) (2) nên f n là hàm cần tìm

Câu 62 Tìm tất hàm số f :  * thỏa mãn điều kiện sau:        ,  1

f nf n  f nf n  n

Lời giải Giả sử có hàm f thoả mãn điều kiện  1

Ta lập dãy  an với n ta đặt anf n   1 trở thành

2 3,

n n n n

aa  a a   n  2

Thay n n phương trình  2 ta an2an4 a an3 n5, n  3

Trừ vế    2 , ta an4anan3an5an1, n

Thay n 0,1,2,3, ta có

 

 

 

4

5

6

a a a a a

a a a a a

a a a a a

  

 

  

 

  

  

Suy a4a0 a a a3 an2an4an , n  4

Ta chứng minh an4 an, nf n  4   f n , n  5

Hay f hàm tuần hoàn với chu kỳ

Thật Giả sử tồn số n0 *mà an04 a n nn0; 

Do an * với  n nên an04an0 1 suy

 

0 0

4 n n n n

aaa a a aa  aa a a a

Do anan2 a an1 n3 với  n , nên với số liên tiếp an01,a an0, n01,an02 phải có

một số lớn

(73)

TẠ P CH Í VÀ TƯ LI ỆU TO ÁN H Ọ C

Suy a4a0  0 a4nan 0 với  nf n 4 f n  với r0,1,2,3trong f

là hàm tuần hoàn chu kỳ

Hàm f xác định ta tính f 0 a f0;  1 a f1;  2 a f2;  3 a3 từ

 2 ta có

1 2

a a a a a a a a

a a a a a a a a a a

   

 

      

 

Mặt khác     

 01  23  30  10 32

1 1

1 1

a a a a a a

a a a a a a

     

 

     



Suy a0 1a2 1 a11a312  6

Do có khả xảy

Khả 1.   

 01  32  13 31 02 02

1 v a v a

3 v a v a

1

a a a a

a a a a                           

Suy raf       0 ;f ;f ;f a a a a0; ; ;1 3  1; 2; 5; , 1; 3; 5; , 5; 2;1; , 5; 3;1; 2      

Ta tìm hàm cần tìm với f n  với f n  xác định

 

   

   

   

   

0 , mod , mod , mod , mod

f n f n f n f n f n            

 7

đều thỏa mà điều kiện

Khả   

 01  32 

1 1

1

a a a a        

 a0 a1 a2 a3  2 a a a a0; ; ;1 3  2; 2; 2; 2

Suy f n( ) 2 với n

Khả   

 01  32 

1

1

a a a a         

Lập luận tương tự khả ta        

f ;f ;f ;f a a a a0; ; ;1 3  2;1; 3; , 3;1; 2; , 2; 5; 3;1 , 3; 5; 2; 1      

Ta hàm f n cần tìm hàm thỏa mãn điều kiện  1 Với f n  xác định  7

Vậy có hàm f thỏa mãn nghiệm phương trình  1

Câu 63 Tìm tất hàm f :    thỏa mãn:

 

     

f x f y  f x y  f y

Lời giải

(74)

CH

INH

PH

C

OL

YM

PIC T

O

ÁN

Thế y y f z   ta được:

 

 

            

f x f y f z   f x f y z   f zf x y  zf y z  f z

f x y f z     f y f z    f x y z    f z f y z   f z 

Thu được: f x y  2z f x y z    f z  với x y z, , 0

Với giá trị y, ta chọn x cho x y f y ,  , từ ta có kết f f y  2f y 

Thế y x vào phương trình ban đầu f x f x    f 2xf x 

Dễ nhận thấy f x x, vậy: f x  f f x   f 2xf x f f x ,    f 2x

Theo phương trình hàm Cauchy, ta f x 2x với x 

Câu 64 Tìm số nguyên dương m nhỏ cho tồn hàm số f : *  \ 1;0;1  thỏa mãn đồng thời điều kiện sau

i) f m  f 2015 , f m1 f2016 ;

ii)    

  11, 1, 2,

f n

f n m n

f n

  

Lời giải

Ta có  

 1     *

2 ,

f n m f n m f n n

f n

       

Với m1, ta có f n 4 f n  f n 4k f n ,k n,  * Ta có  2   1 ; 1    1, *

1

f n

f n f n n

f n f n

      

  1 2015 4.503 3  3  1

1

f f f f

f

      : vô lý

Với m2, ta có f n 8 f n  f n 8k f n ,n k,  *  

  1     *

4 ; ,

1

f n

f n f n n

f n f n

      

Ta có        

 1

2 2015 251.8 7 ;

3

f f f f

f

     

         

         2

1 2016 251.8 8

4

2

2

f f f f

f f

f f f

f

     

     

 Điều mâu thuẫn dẫn đến m3

(75)

TẠ

P

CH

Í VÀ

LI

ỆU TO

ÁN

H

C

Cho a \ 1;0;1 , đặt  1 ;  2 ;  3 1;

a

f a f f

a a

   

  

   

3 ,

1

f n

f n n

f n

   

Khi đó, chứng minh quy nạp hàm số xác định *và f n  \ 1;0;1 ,   n * Hơn theo chứng minh  

 1

6

f n

f n

   ,f n 12k f n ,n k,  *

Khi      

 1   2  

2015 167.12 11 11

5

f

f f f f

f f a

        

2016 167.12 12  12  1   3  4

6

f a

f f f f

f f a

 

       

 

Vậy hàm số thỏa mãn yêu cầu toán

Câu 65 Xác định hàm số f x  liên tục   thỏa mãn đồng thời điều kiện:  f 2x 2f x  với x ,  1

f f x e 3  f x 1x e2 x 1f x 

với x ,  2  f e 1  e 1  f ,  3

f k  số nguyên dương với số nguyên dương k,  4 Lời giải

Với a b,   f a  f b , suy f a e3  f a 1 f b e3  f b 1 Do     

f a 1   3  f b  1

f f a e   f f b e

Hay a e2 a1f a b e2 b1f b  Vì f a  f b 0 nên ta suy a e2 a 1 b e2 b1 Xét hàm số h x x e2 x 1 , ta có h' x 2x ex 1 x e2 x 0 với x  Do hàm số h x x e2 x 1đồng biến 

Do từ a e2 a 1 b e2 b1, ta suy h a h b  hay a b

Vậy f x  đơn ánh Kết hợp với f x  liên tục ta suy f x  hàm đơn điệu thực Mặt khác, theo giả thiết f 2 2 1f  f  1 nên ta suy f x  hàm tăng thực tập 

Từ  2 ta cho x1 f f 3 1 ef 1 1 e 1  f Kết hợp với  3 ta suy f f 3 1 ef 1 1 f e 1

f x  hàm tăng thực  nên ta suy f3 1 ef 1 1 e

(76)

CH

INH

PH

C

OL

YM

PIC T

O

ÁN

Do từ f3 1 ef 1 1 e 1, ta suy g f  1 g 1 hay f  1 1

f  2x 2f x  với x  f 1 1 nên theo quy nạp ta có f 2n 2n với số tự nhiên n

Với số tự nhiên n, ta có

         1

2nf 2nf 2n 1 f 2n2   f 2n2n 1 f 2n 2n

Vì điều kiện  4 nên f2n1 , f 2n2 , ,  f2n2n1 số nguyên dương Do ta suy f 2n 1 2n1,f 2n22n 2, ,f2n2n 1 2n2n1

Vậy f n n với số nguyên dương n

Từ f 2x 2f x  với x  Ta suy f  2nx 2nf x  với x  Cho với

2n

m

x với m n, số nguyên dương ta suy   2

n n

m

f mf  

 

Do hay

2 2

n

n n n

m m m

mf  f  

    số nguyên dương m n, Với x  tùy ý cho trước tồn dãy số  qk , qk có dạng

2n

m

hội tụ đến xf x  hàm liên tục nên xlimkqk limk f q kflimkqk f x 

Thử lại ta thấy hàm số f x x thỏa mãn điều kiện

Câu 66 Tìm tất hàm f:  thỏa mãn đồng thời hai điều kiện sau:  Với cặp a, b ngun dương khơng ngun tố nhau, có

     

f a f bf ab

 Với a, b nguyên dương tồn tam giác không suy biến có độ dài ba cạnh f a f b   , f a b  1

Lời giải

Từ điều kiện  2 , với a, b nguyên dương, ta có

     

     

     

1 ;

1 ;

1 ;

f a f b f a b

f a f a b f b

f a b f b f a

   

 

   

    

 Nếu a b 2: f 4  f  2 ;2 22 f   f  3

Nếu a 3; b :  f 2  f 3  f 4

(77)

TẠ

P

CH

Í VÀ

LI

ỆU TO

ÁN

H

C

Quy nạp chứng minh f n 1 với n nguyên dương

Cho a n b , 2 : f n 1 f n  f  2  2 f n 11

Nếu f 2 2 , quy nạp chứng minh f  2kf  2 2f k1   f  2 k 2k Do f 4  f 2  f 3 2f 2  f  3 3

Quy nạp chứng minh f n   n n,

Cho a n 1,b2 : f n  f n  1 f 2   n f n n Lấy r số nguyên lớn cho 2r không vượt n Nếu 2rn theo chứng minh có f n n

Nếu n2rs với 1 s 2r

Với a n 2rs b;   2r s 1.Ta có

2r 1 2r   2r 1 2r 2r 1

f   s   s f nf   s f     s s

    2r 2r 1 2r 2r 1 2r 1 1  

f nf   f   s     s     s n f nn

Vậy f n   n n,

Do f 1 2 2f 4 nên f  1 1,

Vậy f n 1 với n nguyên dương f n   n n, 2; f   1  1; 2; 3

Câu 67 Tìm hàm số f : 1;   thoả mãn điều kiện:        

f xf yy x f xy với x y, 1 1

Lời giải

Với t > 1, thay x y;      t; , ; 4t 2 ; 2t  vào  1 ta được:        

               

2 2

4 4

2 2

f t f t f t

f t f t f t

f t f t f t

  

 

  

   

  4 3   2 5  4 ,

f t f t t f t t

        2

Lấy  4  2

2

t  ff

Thay vào  2 ta   2 5  4

t f t t f t

    

 

 

Do với 1,  4  2

2

f

t t f t

t

   

Từ  1 ta có f t  f   4 t f  4t 2f 

t

    với 1,

2

tt

Với

t , từ  1 thay 5, 2

(78)

CH

INH

PH

C

OL

YM

PIC T

O

ÁN

    2   2

5

2 5

2

2

f f

f    ff  

    

  2

,

f

f t t

t

  

Đặt c 2f   f x  c

x

   với x1

Thử lại thỏa mãn điều kiện  1 Vậy hàm số cần tìm f x  c

x

Câu 68 Tìm tất hàm f : *  * thỏa mãn đẳng thức:

   

 2  2

f f mf nmn , với m n,  *

Lời giải Nếu m m1, 2 * cho f m 1  f m 2

  2  2   2  2  2 2

1 2 2 2

f f mf nf f mf nmnmn ,

Suy m1 m2 hay f đơn ánh

Từ f m 22f n 2  f p 22f q 2 m2 2n2 p22q2  1

Dế thấy với n *,n3 ta có n222n1 2  n22 2n12  2

Chú ý Điều ta nhân vế với thừa số

Đặt f 1  a f a 3 3 Theo  1 suy ra:

 2  2  2  2  2

5 3 3 27

f af af af af a

Vì phương trình x2 2y2 27 có nghiệm nguyên dương x y;    3,  5,1 nên ta có f a 2 1,f 5a2 5

Cũng từ  1 ta có 2f 4a2 2 2f  a2  f  5a2 2 f a 2 24

Vì phương trình x2y2 12 có nghiệm nguyên dương x y, là 4, 2 nên

 4 4,  2 2

f af a

Từ  1 ta có f k 4a22 2f k 3a222f k 1a22 f ka 2 2, suy từ khai triển  2

Vì theo kết phép quy nạp ta suy f ka 2 k, với k số nguyên dương Do f a 3  a f 1 mà f đơn ánh nên a3   1 a

(79)

TẠ

P

CH

Í VÀ

LI

ỆU TO

ÁN

H

C

Câu 69 Tìm tất số ngun khơng âm n cho tồn hàm f : 0; khác thỏa mãn đồng thời điều kiện sau

i) f xy  f x f y   ,x y, 

ii) 2f x 2y2f x f y x y  ,  0;1; 2; ; n

Lời giải

Với a bất kì, cách thay x y a  k;k * vào i)        

2f a kf f a k   0;1; 2; ;n  1

 Nếu f 2 0 2f a  k 0;1;2; ;n f a 0  Nếu f 2 0 ta thấy f a 0 f a 1

Thật vậy, f a 1 cách cho k , ta thấy 2f a kf   2 f a k1   Nên  1 xảy ra, cịn 0 f a 1 với kđủ lớn ta có

     2 2f a kf f a k   Nên  1 xảy

Thành thử, ta chứng minh với a f a 0 f a 1 Từ suy 2f x 2y2 f x  f y 0;1; ; x y,   2

Do đó, n2

Nếu n0, 2fx2y2 f x  f y ;x y,  Vì f khác nên tồn x0 cho f x 0 0

Khi f x 0  f x f   0  f 1 1

Do f khác nên tồn x1 cho f x 1 1

Từ i), ta có f  0  f x   1 ff  0 0

Bây giờ, sử dụng  2 ta 2 1 f 202 f    1  f 1 Điều vơ lí chứng tỏ n0 khơng thỏa mãn

 Nếu n1 hàm số  

1

0if f x

i x x f

  

 

Thỏa mãn đề Do n1 thỏa mãn đề

 Nếu n2thì ta thấy khơng thể tồn số p q,  ; , p q 1 cho f p 2q20 Thật vậy, trái lại, x y,  ta có

 2 2  2 2  2 2 2 2   2 2

(80)

CH

INH

PH

C

OL

YM

PIC T

O

ÁN

Kết hợp với  2 suy fxp yq  fxp yq0 Thế nhưng,  p q, 1 nên tồn

,

x y để xp yq 1 Do 1 f xp yq  0

Điều vơ lí chứng tỏ fx2y2 1; x y,  ; , x y 1 Bây giờ, ta xét hàm số  

0

p if f x

p x if

q x x q x

 

 

 

   

 

 

, p q, số nguyên tố phân biệt có

dạng 4k3 Ta chứng minh hàm f x  xây dựng thỏa mãn điều kiện:

i) f xy  f x f y   ,x y, 

ii) 2f x 2y2f x f y x y  ,  0;1; 2; ; n

 Kiểm tra điều kiện i)

Nếu p xy

q xy

 

 hiển nhiên f xy  0 f x f y   

Nếu x

pq y pq

 

f xy  1 f x f y     Kiểm tra điều kiện ii)

f x    0;1 nên 2f x 2y2 f x  f y x y  ,  0; 21;  Dễ thấy

     

     

     

2

2

1

2

2

0

2

0

f f p

f p q f p

f p

f q

f f f

  

   

 

 

 

 

 

nên 2f x y2 f x  f y x y  ,  0; 21;  Vậy n1,n2 tất giá trị thỏa mãn đề

Câu 70 Tìm tất hàm số f : * * thoả mãn điều kiện:

 

 2 2 3 2    2    *

2 f mnf m f nf n f m ,m n, 

Lời giải

Giả sử tồn hàm số f thỏa mãn yêu cầu đề

Nếu f n c, với c số hiển nhiên thỏa mãn điều kiện toán Nếu tồn m n,  * cho f m    f n ta gọi a b, số thỏa mãn

    min     , , *

f af bf mf n m n  1

(81)

TẠ P CH Í VÀ TƯ LI ỆU TO ÁN H Ọ C

Từ f a  f b   f a  f b  f a 2b2 f b  f a 2b2 f b   2 Rõ ràng  2 mâu thuẫn với  1

Do f n c, với c số tất hàm cần tìm

Câu 71 Tìm tất hàm số f :  thoả mãn điều kiện:  f 0 c

    

 

3

1 , *

3

f n

f n n

f n

   

  1

Lời giải

Từ  1 ta có  

        tan 1

1 tan

6

f n f n

f n

f n f n

                tan

1 tan f f f      Do ta đặt f 0  c tan  1 tan

6

f   

 

     

tan tan

1 tan 6 6 2

6

2 tan

6

1 tan tan tan

6 6

f f f                                   

Ta chứng minh quy nạp công thức   tan ,

f n    n

   2

Thật vậy, với n0,1,2 công thức  2 Giả sử   tan

6

n

f n    

 

Ta có  

 

   

tan tan

tan 6 6

6

1 tan

6

1 tan tan tan

6 6

n f n

f n n

f n                                  

Hay  2 với n1

Nghiệm toán   tan ,

f n    n

 

Câu 72 Tìm tất hàm số f :  thỏa:

 2      ,

f af bf f a b a b

International Mathematical Olympiad 2019

Lời giải – Trần Bá Đạt

(82)

CH

INH

PH

C

OL

YM

PIC T

O

ÁN

 0     

ff bf f b ,f  2a 2 0f   f f a  , với a b, 

Do f  2a 2f a f  0 , với a

Thay vào phương trình đề ta 2f a 2f b  f  0  f f a b   

Cho a0,b a b  phương trình ta được: f 0 2f a b   f f a b   

Do đó: f a  f b  f a b   f 0

Đặt g x  f x  f 0 suy g a   g bg a b  g x cx hay f x cx d x   Thay vào phương trình ban đầu cân hệ số cho a b ta thu được: c d 0

2

c

Vậy f 0 f x 2x d

Câu 73 Có tồn hay không hàm số f :  cho  

    , ,  1

f m f n  f m  n m n

Lời giải Giả sử tồn hàmf thoả mãn đề

Từ  1 cho m0 ta có f f n   f 0 n  2

Với n n1, 2 mà f n 1  f n 2 f f n  1  f f n  2 

Từ  2 suy f 0 n1  f 0 n2, n1 n2 nên f đơn ánh

Cho n0 từ  1 ta có f m f   0  f m  m f 0 m

Từ ta đượcf  0 0 thay vào  2 có f f n     n n,  3

Từ  1 thay m f m  áp dụng  3 ta f f m   f n   m n

Xét m n p q, , , số nguyên cho m n p q   ,

   

      

f f mf n       m n p q f f pf q

Theo chứng minh f đơn ánh, nên suy f m  f n  f q  f p  Do với n ta có

 1  1      1      1

f n  f n  f nf nf n  f nf nf n

 1      1    2  1  0

f n f n f n f n f f f

          

Nên f n  cấp số cộng với số hạng đầu U1  f 0 0 công sai df  1 suy

ra f n Un1 U1nd nd n , 0

Ta xét với hai số n0,m0 cho m nd 0 thay vào  1  

         

f m f n  f m nd  f m  n f m nd md n  m nd d md n  

(83)

TẠ

P

CH

Í VÀ

LI

ỆU TO

ÁN

H

C

Câu 74 Cho hàm số f :  hàm số thỏa mãn điều kiện sau:

i) f mn  f m f n   ,m n, 

ii)m n  ước f m  f n  với m n, 

Chứng minh tồn số tự nhiên lẻ k cho f n nk, n

India National Olympiad 2018

Lời giải

Gọi P x y ,  phép m x n y ,  vào điều kiện i)

Q x y ,  phép m x n y ,  vào điều kiện ii)

Thế P 1,1 ta f 1.1  f    1 ff 1 1 f  Thế Q2, 2 ta được:

2 2   ff 2 2 f 2  f  2 2 ,kq k , 2, q 1

Giả sử ta xét với q1 tồn số nguyên tố p cho p q suy p số nguyên tố lẻ

Từ f 2 2 ,kq k , 2, q 1 nên ta suy ra: p f  2

Thế 2,

2

p

P  

  ta    

1

2

2

p p

f    f p  f f   

   

 

Từ p f  2 nên ta suy ra: p f p 1  Thế Q1,p1 ta được:

           

1 p1 ff p1 p ff p1 p1 f p 1 p1 Điều hồn tồn vơ lý, ta phải có q 1 f  2 2 k

Thế Q 2,1 ta được:

         

2 1 ff 3 2k 1 2k 1 mod  1 k 1 mod k

phải số lẻ Từ điều kiện i) ta được:

2.2 2      2 2 2 ,k k k

ff f fm lần số

   2m 2k m km f

  

Từ đây, ta Q n , 2m ta được:

       

2m 2m 2m 1km

nf nf  n f n

Mà ta biết rằng: x y x  kykk số lẻ Từ ta suy ra:

    2m k 2m k

nn

(84)

CH

INH

PH

C

OL

YM

PIC T

O

ÁN

 

       

2m 2km k 2km , 2m k,

nf n   n     m n f nn  m

Khi đó, với m số tự nhiên đủ lớn  3 xảy khi:   k, ,

f nn  n k số tự nhiên lẻ Vậy từ ta suy điều phải chứng minh

Câu 75 Tìm tất hàm số f : *  * thỏa mãn đồng thời điều kiện sau:

i) f  0 0,f 1 1

ii) f 0  f  1  f 2 

iii) f x 2y2 f x2  f y2 ,x y,  *

Baltic MO

Lời giải Ta có:

 2 12 12 2 1 2 1 2, 5  12 22 2 1 2 2 5, ,  

n n

ff   ffff   fff xx

Ở đâyx0 1,xn1 x2n  1, n

Hiển nhiên ta có: lim n

xx  

Từ suy f m  f m 1 thì:

 

           

   

2 2 2 2 2 2 2

2

1 1 1 1

1 , 1, 2

f m f m f f m f m f m

f m k f m k m

           

      

Quy nạp lên ta có tồn vơ hạn số m cho f m 2k  f m21 ,  k 1, 2m2 Ta chọn m đủ lớn cho tồn n để a an, n1m2 1,m22m2

Khi anan1 điều hồn tồn vơ lý nên suy f hàm số tăng thực

Từ hiển nhiên ta có: f n   n n, *,thử lại thấy thỏa mãn Vậy tất hàm số thỏa mãn yêu cầu toán f n   n n, *

Câu 76 Tìm tất hàm số f :  thỏa mãm điều kiện sau:

i) Nếu a b f a  f b 

ii) f ab  f a b2 f a  f b ,a b, 

Mathlinks Contest

Lời giải Trước hết, ta có nhận xét nhỏ sau

(85)

TẠ

P

CH

Í VÀ

LI

ỆU TO

ÁN

H

C

Do ta giả sử f 1 0 Chú ý từ 1n f n   0, n Ta giải tốn thơng qua bước sau

Bước Từ f  1.1  f1 1  f 1  f 1 suy f  2  f  1 f  2 0

Bước Gọi n số nguyên cho 1 số phương modulo n Do tồn số a thỏa mãn: a2   1 kn

Suy ra:

   1    1  1    1 0

f af a   f aff a   f knf

Nhưng f n  f kn  f a 21 f n  f 1 nên f n  f 1 0 Do tồn u cho u2  1 mod nf n 0

Bước 3. Từ bước dễ thấy f p 0 với p số nguyên tố p1 mod    Bước Giả sử f a  f b 0 f ab  f a  f b 0 f a 2b20,vơ lý Do f a  f b 0 f ab 0

Bước Gọi a b, hai số nguyên thỏa mãn  a b, 1,khi gọi p ước

2

ab ta có: a2b2 0 mod  p

Mà ta có bổ đề quen thuộc sau Nếu p số ngun tố có dạng 4k3 với bộ số a b, thỏa mãn p a2b2thì ta có p a p b.

Vì  a b, 1 nên p a2b2thì p có dạng 4k1 Từ bước 4 ta có:

2

ab tích số nguyên tố pi thỏa mãn f p i 0 nên f a b20

Mà từ f ab  f a 2b2 f a  f b ,a b,  ta có  a b,  1 f ab  f a  f b   Bước Cho a bc vào phương trình cho ta được:

 2  2     

1

f b cf b c   f bcf b

Nhưng f b  f b c 2 1 f bc  f b c 2 Do ta có: f bc  f b c 2 * 

Thế c1 vào  * được: f b  f b 2 Thế c b vào  * được: f b   2  f b3

Từ đấy, phép quy nạp ta f b kf b , kBước Sử dụng bước 5 bước 6 ta có:

ni  

i i

fp  f p pi số nguyên tố

(86)

CH

INH

PH

C

OL

YM

PIC T

O

ÁN

* f p 0 với số nguyên tố p cho p1 mod 4  p2

* f p ap 0 với số nguyên tố p lại, ap số nguyên không dương

*  ni  

i i

fp f p pi số nguyên tố Ta chứng minh f x  thỏa mãn điều kiện: Hiển nhiên a b f a  f b 

 1.1 1 1  1  1

ff   ff

 .1  1    1

f af a   f af

Ta có ước nguyên tố p a21 thỏa mãn p1 mod   Với hai số nguyên a b, 1 bất kì, ta gọi:

i

p ước nguyên tố a không chia hết cho b

i

q ước nguyên tố b không chia hết cho a

i

r ước nguyên tố a b    i  i

f a f p f r

   i  i

f b f q  f r

   i  i  i

f ab f p f q  f r

 2    ,

i i

f ab f r f s si ước nguyên tố    

2

, ,

a b

A

a b a b

   

   

   

Nhưng tương tự bước 5 ta có:

Các ước nguyên tố A số nguyên tố thỏa mãn si 1 mod 4  f A 0 Suy f a 2b2f r i

Hay ta có f ab  f a 2b2 f a  f b ,a b,  Và ta có nghiệm phương trình hàm là:

Cho M số nguyên, hàm f xác định sau: * f 1 M

* f 2 M

* f p M với số nguyên tố p thỏa mãn p1 mod  

* f p M ap với số nguyên tố p lại, ap số nguyên không dương

*  ni    

i i

(87)

TẠ

P

CH

Í VÀ

LI

ỆU TO

ÁN

H

C

Câu 77 Tồn hay không hàm số f : 1, 2, , n thỏa mãn điều kiện:

i) f hàm đơn ánh

ii) f ab  f a f b  với a b, 1, 2, ,nab n

Lời giải Ta hàm số f sau:

Kí hiệu số nguyên tố bé n theo thứ tự tăng dần là: p p1, , , 2 pk Khi

1

, ,

i

k

i i i

i

a pn

             

1

1 ,i k

i i

i

f p n f a n

   

Ta chứng minh hàm số thỏa mãn yêu cầu toán Thật vậy, với

1

i

k i i

a p



1

i

k i i

b p

 có giá trị ij khác

Thì    

1

1

k i i

f a n

     

1

1

k i i

f b n

 

Hiển nhiên ta biểu thị f a f b   , cách sang hệ số n1 f a  f b 

Mặt khác ta có              

1 1

1

i i

k k k k

i i i i i i i i

i i i

i

f ab f p  f p f p f p f a f b

  

 

           

    

Hay ta có: f ab  f a  f b , theo giả thiết đề

Vậy từ hàm số xây dựng thỏa mãn yêu cầu tốn

Câu 78 Giả sử Josephus có n1 người bạn, n người thành vòng tròn đánh số từ đến n theo chiều kim đồng hồ, tự sát theo nguyên tắc, người thứ cầm dao đếm tự sát, người thứ hai đếm tự sát,…Q trình dừng lại cịn người Gọi f n  hàm số biểu thị vị trí cùa người sống sót Câu hỏi đặt là, tính f n  ?

Bài tốn cổ Josephus

Lời giải Ta xét hai khả n chẵn lẻ

Khả 1. Khi số n số chẵn, tức n2 k Sau vịng cịn người vị trí lẻ Số người đánh lại thành 1, 2, , k

Nếu lượt trước người có số 2i1 sau mang số i Người sống sót có số cũ  2

f k sau mang số f k  Vậy từ ta có f 2k 2f k 1

(88)

CH

INH

PH

C

OL

YM

PIC T

O

ÁN

Nếu lượt trước người có số 2i1 sau mang số i Người sống sót có số cũ 2 1

f k sau mang số f k  Vậy từ ta có f2k12f k 1 Như thìf  1 1, 2f k 2f k 1, 2fk12f k 1

Ta chứng minh quy nạp

Nếu biểu diễn số n là: na ak k1 a1 2 ,ak 1,với i kai 0,1

Thì f n   ak1 a a1 k2 Thật vậy:

Với n1 ta thấy hiển nhiên

Giả sử với k n mệnh đề Ta xét hai trường hợp sau đây:

Trường hợp Nếu n số chẵn, đặt n2 m

Khi như: mb bk k1 b1 2 2mb bk k1 0b1 2

f 2m 2f m  1 2bk12k  b1.2 1   bk1 01b1 2

Vậy trường hợp 1 mệnh đề

Trường hợp Nếu n số lẻ, đặt n2m1

Khi như: mb bk k1 b1 2 2m 1 b bk k1 1b1 2 Và f2m12f m  1 2bk12k  b1.2 1   bk1 11b1 2

Vậy trường hợp 2 mệnh đề

Từ đó, theo ngun lý quy nạp mệnh đề ban đầu từ ta suy điều phải chứng minh

Câu 79 Cho hai hàm số f g, : *  * hai hàm số thỏa mãn đồng thời điều kiện:

i) g hàm số toàn ánh

ii) 2f n2 n2g n2 , n

Nếu f n  n 2019 n n,   f có vô số điểm bất động Lời giải

Đầu tiên ta có định lý Dirichlet số nguyên tố dãy số  pi với pi số ngun tơ

có dạng 8k3 dãy vơ hạn

Từ với n, theo cơng thức kí hiệu Legendre ta có:   281

2

1 pn

n p

 

   

 

 

Sử dụng điều kiện i) ta tìm dãy  xn n1 cho g x npn, n  Ta có 2f x2 nxn2pn2 2f x2 nx2nmodpn

n p

 

 

 

  nên suy ra:

 

n n n p f n p x

 

(89)

TẠ

P

CH

Í VÀ

LI

ỆU TO

ÁN

H

C

Suy tồn hai dãy số nguyên dương  an  bn cho  

n n n

n n n

x a p

f x b p

 

 

 Từ điều kiện ii) ta 2bn2 a2n1

Cuối cùng, sử dụng giả thiết: f n  n 2019 n n,  thì ta có:  

2019

1

n n

n n

n

f x b

x a

x    

2 1

lim n lim

n

x x

n

a

a a

 

   

Suy tồn N0 cho anbn   1, n N0

Vậy từ f p npn, n N0

Và từ ta suy điều phải chứng minh

Câu 80 Tìm tất hàm số g: *  * thỏa mãn điều kiện sau:  

     

g g nn g n1   3 n g n , n

Doãn Quang Tiến

Lời giải

Đặt g n  f n   n n, * thay vào phương trình hàm ban đầu ta được:  

           

     

* *

1 ,

1 2,

f f n f n f n n n f n n n

f f n f n n n

          

      

Từ đây, ta chuyển toán ban đầu thành tốn khác gọn đẹp nhiều

Thay n1 vào  1 ta f f  1  f  2 3

Từ ta suy ra: f  2 2 f f  1 2 Từ ta xét hai trường hợp sau:

Trường hợp 1. f 2 1 f f  1 2 Bây ta đặt f 1  k f k 2

Thay n2 vào  1 ta đượcf f  2  f  3 4 Từ suy raf  3  4 f 1  4 k

Từ f 3 1 nên suy ra: k3

Nếu k1 ta có: 2 f f  1  f k  f  1 1,điều mâu thuẫn Nếu k2 ta có: 2 f f  1  f k  f 2 1,điều mâu thuẫn

Nếu k3 ta có: 2 f f  1  f k  f  3     4 k 1,điều mâu thuẫn Vậy tóm lại khơng có giá trị k thỏa mãn nên trường hợp 1 không xảy

Trường hợp f 2 2 f f  1 1

Thay n2 vào  1 ta đượcf f  2  f  3 4

(90)

CH

INH

PH

C

OL

YM

PIC T

O

ÁN

                    

4 5

5 6

6 7 4

f f f f

f f f f

f f f f

    

    

    

Từ ta dự đoán rằng, hàm số f n  xác định sau:

  1, *

f n       n n n   

Bây ta chứng minh rằng, hàm số hàm số thỏa mãn  1 để từ suy cơng thức hàm g n  từ ta hồn tất toán

Mà trước tiên, để chứng minh nhận định đó, ta cần phải có hai bổ đề sau:

Bổ đề Với số n * ta có  1

1

n

n n

n

      

   

Chứng minh

Trước hết ta có     n n 1     n n 1     n 1    n n Và     n n 1              n n 1 n 1 n

Do lưu ý rằng: 1

2

  

      nên suy ra:    11 Vậy từ ta thấy bổ đề 1 chứng minh

Bổ đề Với số n * ta có  1  1

1

n if n n n

n

n otherwise

       

     

  

  

 

 

 

  

Chứng minh

Hiển nhiên ta có n1 n1 n2 Giả sử n1     n từ ta có:

n n 1   n 1 n n 1 n n 1 1 n

                        

   

Và ta có     n n 1     n n 1     n 1    n n Từ ta suy được:     n n 1 n

Giả sử n1     n từ ta có:

n n 1   n 1 n 1 n 1 n 1 n

                   

   

Tử theo bổ đề 1 ta thu được:     n n 1n Vậy từ bổ đề 2 chứng minh

Quay trở lại với việc giải toán

(91)

TẠ

P

CH

Í VÀ

LI

ỆU TO

ÁN

H

C

Với n2 f  2   2  2

Giả sử kết với 1 j n Sử dụng  1 ta có:

 1     1  1  1

f n   n f f n   n f   n n   n    n n      n n

Mà từ    n n 2n n   1 n ta có f n 1        n   n n 1 Giả sử n thỏa mãn:     n n 1n từ ta có: n1     n Và ta suy f n 1  n1n

Nếu n không thỏa mãn     n n 1n tức xảy ra:

n n 1 n       

 

Và theo bổ đề thì ta được: n1     n Và từ ta suy được:

 1  1  1  1

f n         n   n n     n n       n n  n n

Vậy từ theo nguyên lý quy nạp mệnh đề  1 chứng minh hoàn toàn Từ suy tất hàm số thỏa mãn  1

  1, *

f n       n n n    Hay từ ta suy hàm g n  mà cần tìm là:

    1 1, *

g nf n           n  n n n  n n   

Câu 81 Cho ba số thực a b c, , không âm, phân biệt cho tồn hàm f g, :  thỏa mãn af xy  bf x cf x  g y 

y

 

   

  với số thực dương x y Chứng minh tồn hàm h:  cho:

  x    ,

f xy f f x h y x y

y

 

      

 

Iran TST 2019

Lời giải

Đặt P x y ,  phép cho phương trình: af xy  bf x cf x  g y 

y

 

   

 

Trường hợp 1: Nếu a b c  0: Ta có P x ,1 f x  g 1

a b c

 

  số, vậy:    

x

f xy f f x

y

 

  

(92)

CH

INH

PH

C

OL

YM

PIC T

O

ÁN

Trường hợp 2: Nếu a b c  0:

Ta có a f xy   f x  b f x f x  g y 

y

   

     

 

  đặt phép Q x y , 

Lấy Q 1,yQ x y ,  ta

       

  x    1  1

a f xy f x f y f b f f x f f

y y

     

          

   

 

Thế y

y

 vào  1 ta được:

   1         1 

x

a f f x f f b f xy f x f y f

y y

     

       

     

     

 

Do a b , a b c  0 a2b2 0, vậy:

       1

f xyf xf yf ,x y, 0

   1

x

f f x f f

y y

   

  

   

    ,x y, 0

Cộng hai phương trình với nhauf xy  f x 2f x  f y  f 1f 

y y

   

      

   

Vậyh y  f y  f 1f  y

y

 

    

  , thỏa mãn

Câu 82 Tìm tất hàm số f :  thỏa mãn:      

! ! ! ! , ,

nf m f nf mm n

BMO Shortlist 2018

Lời giải Đặt phép P m n ,  cho phương trình ban đầu

 1,1

P  1 f   1 ! ff 1 !  f 1 ! 1 f 1 1, từ hiển nhiên f 1 1  1, !  !  ! !  

P n  n f n   f nnf nn

Gọi p số nguyên tố tùy ý, ta có P1,p 1 p1 ! 1 f p 1 ! 1 Theo định lý Wilson số học ta đượcp p 1 ! 1  , suy p f p 1 ! 1 

(93)

TẠ

P

CH

Í VÀ

LI

ỆU TO

ÁN

H

C

Với giá trị m bất kỳ, ta chọn p đủ lớn để thu được: f m !  f m !, sử dụng kết ta được: n! f m f n   ! ! f m ! tương đương n! f m   ! f n !n!

Thay m p 1 với p đủ lớn vào phương trình f n !n! với n Vậy f n n hàm số cần tìm

Câu 83 Tồn hay khơng hàm số f : *  * thỏa mãn điều kiện sau:  

  3 2  , *

f f nnf n  n

Lời giải Giả sử tồn hàm số f thỏa mãn yêu cầu toán

Với i * ta xây dựng dãy số sau  an n1 cho: a1 i a, n1  f a n

Khi ta có an1  f a nf f a  n12f an13an1 2an3an1

Hay ta có an44an13an   0, n

Do an   0, n nên đẳng thức xảy

Nên từ ta kết luận khơng tồn hàm số f thỏa mãn yêu cầu toán

Câu 84 Tìm tất hàm số tăng thực f : *  *thỏa mãn điều kiện sau:  

  2  , *

f n f n  f n  n

Lời giải Do f hàm số tăng thực nên ta có:

 1  

f n  f n  hay f n    1 n f n n Suy ra: f n n hàm số tăng

Mặt khác ta đặt a0 1, an1 anf a n

Từ ta suy a0 a1  f an12f a n

Do f an1an1  f a nan, n *

Suy có vơ hạn số m n,  cho: f m  m f n n Suy f n  n c, với c số

Vậy tất hàm số thỏa mãn yêu cầu toán là: f n    n c n, *,c số

Câu 85 Tìm tất tồn ánh f :  cho với m n,  thỏa mãn:    

f m f nm n

Lời giải Kí hiệu P tập tất số nguyên tố Xét đơn ánh g P: P

1

i

k i i

n p

    

1

i

k i i

f n g p

(94)

CH

INH

PH

C

OL

YM

PIC T

O

ÁN

Kí hiệu  n số ước nguyên dương n

Mà ta có nhận xét sau:  nf  nf toàn ánh

Với số nguyên tố p f p  có hai ước nguyên tố nên số nguyên tố

Xác định hàm g từ ta có: f p g p  nên ta chứng minh g song ánh Thật vậy, f toàn ánh nên g toàn ánh nên g song ánh

Tiếp theo, ta chứng minh f p kg p k với k số nguyên dương quy nạp Ta thấy rằng, với k1 hiển nhiên

Giả sử mệnh đề với k1

Ta có f p k chia hết cho 1,g p g p   , 2, ,g p k1 ngồi khơng chia hết cho số nguyên dương khác

Do f p k   pk  k

Nếu k1 f p k có thêm ước nguyên tố f p k 2k k 1,vơ lý Từ f p k lũy thừa g p  có k1 ước nên f p kg p k

Giả sử n số nguyên dương, p số nguyên tố không chia hết cho n Bây ta chứng minh f n f p     kf npk , k

Từ n p, k1 nên ta có:  n    pk   npk Mặt khác g p k f np k g p f n   

Do ước f n  g p k chia hết cho f np k ước g p k f n  ước f np k

Lại có f n f p   k   npk  f np k

Nếu f np k có ước khác với ước f n  g p k    

k    k ,

f n f p f np

   vơ lý

Vậy từ ta có kết f np kf n g p   kf n f p   k

(95)

TÀI LIỆU THAM KHẢO

Dưới tài liệu mà ebook có tham khảo đồng thời có tài liệu mà bọn đề xuất cho bạn đọc

[1] Nguyễn Văn Mậu (1997), 00 Phương trình hàm 00, NXB Giáo dục

[2] Nguyễn Trọng Tuấn (2004), “Bài toán hàm số qua kì thi Olympic”, NXB Giáo dục

[3] Nguyễn Tài Chung, Lê Hồnh Phị (2013), “Chun khảo phương trình hàm” Nhà xuất Đại học quốc gia Hà Nội

[4] J.Aczél (1966), “Lectures on functional equations and their applications”, ACADEMIC PRESS New York San Francisco London

[5] Stevo Stevic (2004), “Periodic Character of a Class of Difference Equation”, Taylor & Francis Group

[6] Valentine Boju, Luis Funar - The Math Problems Notebook

[7] Titu Andreescu, Razvan Gelca – Birkhauser Mathematical Olympiad Challenges [8] Edward Lozansky , Cecil Rousseau – Winning Solutions

[9] The IMO Compendium A Collection of Problems Suggested for The International Mathematical Olympiads: 1959 – 2009 – Djukic D., Vladimir Jankovic, Ivan Matic, Nikola Petrovic – Springer (2011)

[10] Trần Nam Dũng, Dương Bửu Lộc – Chuyên đề Phương trình hàm tập số nguyên [11] (Developments in Mathematics 39) Saïd Abbas, Mouffak Benchohra – Advanced [12] Functional Evolution Equations and Inclusions-Springer International Publishing (2015) [13] Aczel – Lectures on functional equations and their applications – Academic Press (1966) [14] An Introduction to the Theory of Functional Equationsand Inequalities – Marek Kuczma [15] Analytic Solutions of Functional Equations – Sui Sun Cheng, Wenrong Li

[16] Functional Analysis, Sobolev Spaces and Partial Differential Equations – Haim Brezis [17] Topics in Algebra and Analysis Preparing for the Mathematical Olympiad –Radmila [18] Bulajich Manfrino, José Antonio Gómez Ortega, Rogelio Valdez Delgado-Birkhäuser Basel (2015)

[19] Kỷ yếu gặp gỡ toán học kỷ yếu từ hội thảo chuyên đề tỉnh [20] The art of Mathematics

[21] 101 Problems in Algebra from the training of the USA IMO team – T Andreescu, Z Feng

[22] Problem Primer for the Olympiad – C R Pranesachar, B J Venkatachala, C S Yogananda

(96)(97)

TẠP CHÍ VÀ TƯ LIỆU TỐN HỌC

Thơn – Thạch Hòa – Thạch Thất – Hà Nội Điện thoại: 0343763310; Email: tuangenk@gmail.com Fanpage: https://www.facebook.com/OlympiadMathematical/

CHỊU TRÁCH NHIỆM NỘI DUNG DOÃN QUANG TIẾN

NGUYỄN MINH TUẤN TÔN NGỌC MINH QUÂN

BIÊN TẬP NGUYỄN MINH TUẤN TRÌNH BÀY BẢN THẢO

NGUYỄN MINH TUẤN LA THỊ ĐƠNG PHƯƠNG

PHƯƠNG TRÌNH HÀM TRÊN TẬP RỜI RẠC

(98)

0343763310

tuangenk@gmail.com Lovetoan.wordpress.com Đại học FPT Hà Nội

CHINH PHỤC OLYMPIC TOÁN

NGUYỄN MINH TUẤN

MỌI Ý KIẾN THẮC MẮC XIN VUI LÒNG GỬI VỀ ĐỊA CHỈ

] The art of Mathematics. ] https://artofproblemsolving.com e: https://www.facebook.com/OlympiadMathematical/

Ngày đăng: 20/02/2021, 23:00

TỪ KHÓA LIÊN QUAN

w